Finman Working Capital [PDF]

  • 0 0 0
  • Gefällt Ihnen dieses papier und der download? Sie können Ihre eigene PDF-Datei in wenigen Minuten kostenlos online veröffentlichen! Anmelden
Datei wird geladen, bitte warten...
Zitiervorschau

MANAGEMENT ADVISORY SERVICES

Working Capital Management

WORKING CAPITAL MANAGEMENT

D. Short-term notes payables are retired with cash.

Theories:

7.

Tiger, Inc. has current ratio of 0.95 is to 1.00. Which of the following would raise the company’s current ratio? A. Declaration of cash dividend. B. Payment of accounts payable. C. Collection of accounts receivable D. Purchase merchandise in a 2/10, net 30 open account. Bobadilla

8.

Which one of the following transactions would increase the current ratio and decrease net profit? A. A stock dividend is declared. B. Vacant land is sold for less than the net book value. C. An income tax payment due from the previous year is paid. CMA 0697 D. Uncollectible accounts receivable are written off against the allowance account.

9.

If a firm increases its cash balance by issuing additional shares of common stock, working capital A. increases and the current ratio increases. B. increases and the current ratio decreases. C. increases and the current ratio remain unchanged. D. remains unchanged and the current ratio remains unchanged. CMA*

Working Capital Policy 1. Net working capital is the difference between A. Current assets and current liabilities C. Shareholders investment and cash B. Fixed asset and current liabilities D. Total assets and total liabilities CMA 0692 2.

3.

4.

5.

6.

Working capital is important for all the following reasons except that it: A. affects a firm’s liquidity and profitability. B. consists of a large portion of a firm’s total assets. C. consists of those assets that are most manageable. D. consumes a small portion of the financial manager’s time.

Cabrera

Which of the following statements is correct? A. Working capital is a measure of long-term solvency B. The stockholder’s equity is a major component of working capital Roque 2013 C. Net working capital is the difference between quick assets and current liabilities. D. Net working capital is the difference between current assets and current liabilities. The longer the firm’s accounts payable period, the: A. Shorter the firm’s inventory period. B. Longer the firm’s cash conversion period. C. Less the firm must invest in working capital D. More the delay in the accounts receivable period.

Bobadilla

Starrs has current assets of 300,000 and current liabilities of 200,000. Starrs could increase it's working capital by the A. Prepayment of 50,000 of next year's rent B. Refinancing of 50,000 of short-term debt with long -term debt C. Purchase of 50,000 of financial assets held for trading for cash D. Acquisition of land valued at 50,000 through the issuance of common shares CMA 1293 Which one of the following transactions does not change the current ratio and does not change the total current assets? A. A cash dividend is declared. B. A fully depreciated asset is sold for cash. C. A cash advance is made to a divisional office.

CMA 0697

10. Working capital management involves investment and financing decisions related to: A. sales and credit. B. current assets and capital structure. C. current assets and current liabilities. D. plant and equipment and current liabilities. Bobadilla 11. The primary objective of working capital management is to: A. achieve a balance between risk and return. B. maximize the company’s total current assets. C. minimize the company’s total current liabilities. D. balance the amount of current assets and current liabilities.

Roque 2013

12. Which of the following assumptions does not underlie risk-return tradeoffs in managing working capital? A. Fixed assets remain constant. B. The yield curve is downward sloping. Page 1 of 37

MANAGEMENT ADVISORY SERVICES

C. Current assets are less profitable than fixed assets. D. Short-term financing is less expensive than long-term financing.

Working Capital Management

Cabrera

13. Which of the following is incorrect A. Profitability varies directly with liquidity. B. The greater the risk, the greater is the potential for larger return. C. More current assets lead to greater liquidity, but yield lower returns. D. Long-term financing has less liquidity risk than short-term financing, but has a higher explicit cost, hence lower return. Roque 2011 14. Which of the following statements is false? A. Liquidity is the ability to convert an asset into cash without significant loss. B. Lengthening the cash cycle increases a firm’s required level of working capital. C. Working capital management uses only a small portion of the financial manager’s time. D. The goal of working capital management is to maintain the optimal level of net working capital. Cabrera 15. Which of the following statement is false? A. Net working capital equals working capital less current liabilities. B. A firm with a current ratio greater than one has positive net working capital. C. Working capital management concerns decisions about all of a firm’s assets. Cabrera D. Net working capital is that portion of a firm’s current assets financed with long-term funds. 16. The optimal level of working capital depends on all of the following factors except the: A. Kind of firm. C. Stability of dividends. B. Length of the cash cycle. D. Variablity of cash flows. 17. Determining the appropriate level of working capital of the firm requires A. Changing the capital structure and dividend policy of the firm. B. Offsetting the profitability of current assets and current liabilities against the probability of technical insolvency. C. Evaluating the risk associated with various levels of fixed assets and the types of debt used to finance those assets. D. Maintaining a high proportion of liquid assets to the total assets in order to maximize the return on total investment. RPCPA 0596 18. A firm following an aggressive working capital strategy would: A. hold substantial amounts of liquid assets.

B. minimize the amount of short-term financing. C. finance fluctuating assets with long-term financing. D. minimize the amount of funds held in liquid assets.

Cabrera

19. Conservative working capital management strategies involve: A. high risk, high return. C. low risk, low return. B. low risk, high return. D. moderate risk, moderate return.

Cabrera

20. Compared to other firms in the industry, a company that maintains a conservative working capital policy will tend to have a A. Higher total asset turnover B. Greater percentage of short-term financing C. Higher ratio of current assets to fixed assets D. Greater risk of needing to sell current assets to repay debt RPCPA 0595 21. As a company becomes more conservative in working capital policy, it would tend to have a(an) A. Decrease in its acid-test ratio. B. Increase in the ratio of current assets to units of output. C. Increase in the ratio of current liabilities to non-current liabilities. D. Increase in funds invested in common stock and a decrease in funds invested in marketable securities. CMA 1296 22. Temporary working capital supports A. acquisition of capital equipment. B. payment of long term debt.

C. seasonal peaks. Bobadilla D. the cash needs of the company.

23. According to the hedging approach, working capital should be financed with: A. long-term financing. C. short-term and long-term financing. B. short-term financing. D. spontaneously generated funds. Cabrera 24. The hedging approach to financing involves A. The use of long-term debt to finance current assets. B. Matching maturities of debt with specific financing needs. C. The use of short-term debt to finance non-current assets. D. Issuance of common stocks to raise funds for working capital requirements. Roque 2011 25. The financing of the basic level of current assets by issuing commercial paper is inconsistent with Page 2 of 37

MANAGEMENT ADVISORY SERVICES

A. B. C. D.

the maximization of shareowners' wealth the goal of minimizing the cost of debt financing the objective of matching the maturities of assets and liabilities the expectation that long-term interest rates will decrease the coming year

Working Capital Management

the former will maximize the return on total assets. Cabrera

Roque 2011

31. Short-term financing plans with high liquidity have: A. high return and high risk C. moderate return and moderate risk B. low profit and low risk D. none of the given choices Bobadilla

26. An advantage of the use of long-term debt as opposed to short-term debt to finance current assets is A. It is easy to repay B. It decreases the risk of the firm C. It generally is less costly than short-term debt D. It generally places fewer restrictions on the firm Wiley 2012

32. Which of the following would increase risk? A. Raise the level of working capital. B. Increase the amount of equity financing. C. Increase the amount of short-term borrowing. D. Decrease the amount of inventory by formulating an effective inventory policy. Bobadilla

27. Financing inventory build-up with long-term debt is an example of A. Matching policy. B. Hedging policy. C. An aggressive working capital policy D. A conservative working capital policy.

33. When a firm finances long-term assets with short-term sources of funding, it A. Improves the leverage ratio B. Will have higher interest expense C. Reduces the risk of cash shortage D. Is ignoring the principle of matched maturities

Roque 2011

28. In a conservative or relaxed working capital financing policy, A. Operations are operated with too much working capital. B. Operations are conducted on a minimum amount of working capital. C. The company is exposed to risk of illiquidity because of low working capital position. D. Short term liabilities are used to finance not only temporary current assets, but also part or all of the permanent current asset requirements. Roque 2011 29. As a company becomes more conservative with respect to working capital policy, it would tend to have a(n) A. Increase in the operating cycle. B. Decrease in the operating cycle. C. Increase in the ratio of current assets to current liabilities. D. Increase in the ratio of current liabilities to noncurrent liabilities. Bobadilla 30. Which of the following statements is true? A. Short-term debt is usually more expensive than long-term debt. B. A conservative working capital policy is characterized by higher current ratio and acidtest ratio. C. Determining the appropriate level of working capital for a firm requires changing the firm’s capital structure and dividend policy. D. Liquid assets do not ordinarily earn higher returns relative to long-term assets, so holding

34. The probability of technical insolvency is reduced by: A. maintaining a high level of liquid assets. B. financing fluctuating assets with long-term debt. C. financing permanent assets with short-term debt. D. both A and B.

Bobadilla

Cabrera

35. If the firm was to shift P2,000 of current liabilities to long term fund, the firm's net working capital would____ , the annual cost of financing would _____, and the risk of technical insolvency would____, respectively A. decrease, decrease, increase C. increase, decrease, decrease B. decrease, increase, decrease D. increase, increase, decrease 36. Zap Company follows an aggressive financing policy in its working capital management while Zing Corporation follows a conservative financing policy. Which one of the following statements is correct? A. Zap has less liquidity risk while Zing has more liquidity risk. B. Zap has a low current ratio while Zing has a high current ratio. C. Zap has low ratio of short-term debt to total debt while Zing has a high ratio of short-term debt to total debt. D. Zap finances short-term assets with long-term debt while Zing finances short-term assets with short-term debt. Bobadilla Page 3 of 37

MANAGEMENT ADVISORY SERVICES

Working Capital Management

A. Accounts payable deferral period B. Accounts receivable period

C. Inventory conversion period D. Operating cycle

37. Which of the following statements is false? A. Using short-term debt to finance permanent assets increases the risk of insolvency. B. Financing fluctuating current assets with long-term financing is a conservative strategy. C. The term permanent assets refers only to fixed assets such as machinery, buildings, and equipment. D. Maintaining a high level of current assets in the form of marketable securities reduces the probability of technical insolvency. Cabrera

44. As a firm's cash conversion cycle increases, the firm: A. becomes less profitable B. incurs more shortage costs C. reduces its accounts payable period D. increases its investment in working capital

38. Which of the following statements is true? A. A higher level of working capital increases the firm’s profitability. B. Long-term financing is used to finance current assets under the hedging approach. C. Technical solvency is the inability of a firm to pay its obligations as they come due. D. The hedging approach is an example of an aggressive working capital management strategy. Cabrera

45. Ignoring cost and other effects on the firm, which of the following measures would tend to reduce the cash conversion cycle? A. Take discounts when offered B. Forgo the discounts that are currently being taken C. Maintain the level of receivables as sales decrease D. Buy more raw materials to take advantage of price breaks Brigham

39. All of the following statements in regard to working capital are correct except A. Profitability varies inversely with liquidity. B. Current liabilities are an important source of financing for small firms. C. The hedging approach to financing involves matching maturities of debt with specific financing needs. D. Financing permanent inventory build up with long-term debt is an example of an aggressive working capital policy CMA 0696

46. Which of the following actions is likely to reduce the length of a firm’s cash conversion cycle? A. Reducing the amount of time the firm takes to pay its supplier. B. Increasing the average days sales outstanding on its accounts receivable. Wiley 2012 C. Adopting a new inventory system that reduces the inventory conversion period. D. Adopting a new inventory system that increases the inventory conversion period.

Cash & Marketable Securities Management 40. The length of time it takes for the initial cash outflows for goods and services to be realized as cash inflows from sales is called A. Cash conversion cycle C. Product life cycle B. Manufacturing cycle D. Vicious cycle Roque 2011 41. The length of time between payment for inventory and the collection of cash is referred to as: A. cash conversion cycle C. payables deferral period B. operating cycle D. receivables conversion period Bobadilla 42. The average length of time a peso is tied up in current asset is called the: A. cash conversion cycle. C. net working capital. B. inventory conversion period. D. receivables conversion period.

Bobadilla

43. The length of time between the acquisition of inventory and payment for it is called the

Wiley 2012

Bobadilla

47. If everything else remains constant and a firm increases its cash conversion cycle, its profitability will likely A. Decrease C. Increase if earnings are positive B. Increase D. Not be affected Wiley 2012 48. An increase in sales resulting from an increased cash discount for prompt payment would be expected to cause a(n): A. increase in the operating cycle. B. decrease in the cash conversion cycle. C. increase in the average collection period. D. decrease in the purchase discount taken. Cabrera 49. An objective of cash management is to A. Maximize the cash balance to avoid the risk of illiquidity. B. Minimize the cash balance to maximize the return from idle cash. C. Reserve as much cash as possible for potential investment opportunities. Roque, 2011 D. Invest cash for a return while retaining sufficient liquidity to satisfy future needs. Page 4 of 37

MANAGEMENT ADVISORY SERVICES

Working Capital Management

50. All of the following are valid reasons for a business to hold cash and marketable securities except to A. maintain a precautionary balance. B. satisfy compensating balance requirements. C. earn maximum returns on investment assets. D. maintain adequate cash needed for transactions. Cabrera 51. The transaction motive for holding cash is the A. A safety cushion C. Daily operating requirements B. Compensating balance requirements D. None of the given choices

Bobadilla

52. A typical objective sought in the effective management of a company’s cash could be expressed as follows: A. To minimize the corporate investment in the accounts receivable. B. To attain that level of profit margin per sales pesos and receivables turnover that maximizes sales. C. To provide the means of paying off accounts when due and thereby helping to maintain the firm’s credit rating. D. To coordinate the activities of the manufacturing and the marketing areas so that the corporation can maximize its profits. Cabrera 53. A precautionary motive for holding excess cash is A. To enable a company to have cash to meet emergencies that may arise periodically B. To enable a company to meet the cash demands from the normal flow of business activity C. To enable a company to avail itself of a special inventory purchase before process rise to higher levels. D. To avoid having to use the various types of lending arrangements available to cover projected cash deficits. RPCPA 0595 54. The risk in our economy which is typically associated with the holding of cash is referred to as A. business risk. C. market rate risk. B. market risk. D. purchasing power risk. Cabrera 55. The difference between the cash balance on the firm's books and the balance shown on the bank statement is called: A. a float C. the compensating balance B. a safety cushion D. none of the given choices Bobadilla

56. Which of the following is true about a firm’s float? A. A firm strives to minimize the float for both cash receipts and cash disbursements. B. A firm strives to maximize the float for both cash receipts and cash disbursements. C. A firm strives to maximize the float for cash disbursements and minimize the float for cash receipts. D. A firm strives to maximize the float for cash receipts and minimize the float for cash disbursements. Wiley 2012 57. Which of the following actions would not be consistent with good management? A. Minimizes the use of float. B. Increased synchronization of cash flows. C. Use of checks and drafts in distributing funds. D. Maintaining an average cash balance equal to that required as a compensating balance or that which minimizes total cost. RPCPA 0595 58. The following practices will impact the cash flow of the company: 1. Sales personnel are unequivocally responsible for collecting their credit sales 2. Sales commissions are based on collected invoices 3. Statement of accounts receivable are reconciled with customers and regularly sent for confirmation 4. Automatic transfer of funds is arranged with banks regarding deposits of branches. Of the above, which will result to a better cash flow? A. Statement 4 only C. Statements 1, 3 and 4 only B. Statements 3 and 4 only D. All Statements RPCPA 0594* 59. The collection of accounts receivable can be accelerated by the use of A. a lockbox system C. remittance advices B. bank drafts D. turnaround documents

Cabrera

60. Which of the following cash management techniques focuses on cash disbursements? A. Depository transfer checks C. Preauthorized checks B. Lockbox system D. Zero-balance account Bobadilla 61. A working capital technique that increases the payable float and therefore delays the outflow of cash is A. A draft C. Concentration banking CMA 1296 B. A lockbox system D. Electronic Data Interchange (EDI) Page 5 of 37

MANAGEMENT ADVISORY SERVICES

Working Capital Management

62. A method of delaying the disbursement of cash by a corporation with a liquidity problem would be to A. install a lock box program B. utilize a concentration banking program C. take as many cash discounts as possible D. pay its bills through the use of bank drafts Cabrera 63. An automated clearing house (ACH) electronic transfer is a(an) A. Computer-generated deposit ticket verifying deposit of funds B. Electronic payment to a company’s account at a concentration bank C. Check like instrument drawn against the payer and not against the bank. D. Check that must be immediately cleared by the Banko Sentral ng Pilipinas

CMA 0694

64. Which of the following is true about electronic funds transfer from a cash flow standpoint? A. It is never beneficial from a cash flow standpoint B. It is always beneficial from a cash flow standpoint C. It is beneficial from a cash receipts standpoint but not from a cash disbursements standpoint D. It is beneficial from a cash disbursements standpoint but not from a cash receipts standpoint Bobadilla 65. The primary reason financial analysts often focus on net cash inflows rather than profits when evaluating company performance is that A. net cash inflows are not subject to distortion. B. net cash inflows indicate the company’s liquidity C. net cash inflows measure the present value of profits. D. net cash inflows are subject to less distortion than profits Cabrera 66. The cash budget is one of the prime instruments used to aid in the cash management process and A. Is basically a longer term forecasting mechanism. B. provides a quick method of projecting profits for manufacturing firms. C. Is rather inflexible and consequently, cannot be adjusted very well for risk. D. Aids in determining when cash deficits and cash surpluses are expected to arise within the budget period. Cabrera 67. The most direct way to prepare a cash budget for a manufacturing firm is to include A. projected sales, credit terms and net income. B. projected net income, depreciation and goodwill amortization.

C. projected purchases, percentages of purchases paid, and net income. D. projected sales and purchases, percentages of collections, and terms of payments. CMA* 68. For a manufacturing firm, the most direct way of preparing a cash budget requires incorporation of the following, except A. sales projections and credit terms. B. collection percentages and other cash receipts. C. projected net income and depreciation expenses. D. estimated purchases and payment terms and other cash disbursements. Roque, 2011 69. Franklin Inc., is a medium-size manufacturer of toys that makes 25% of it sales to Mel Company, a major national discount retailing firm. Mel will be requiring Franklin and other suppliers to use Electronic Data Interchange (EDI) for inventory replenishment and trade payment transactions as opposed to the paper-based systems previously used. Franklin would consider all of the following to be advantages using EDI in its dealings with Mel except A. Reduction in payment float B. Better status of deliveries and payment C. Access to Mel’s inventory balances of Franklin’s products. D. Compatibility with Franklin’s other procedure and systems Agamata 2013 70. When a company is evaluating whether the ratio of cash and marketable securities to total assets should be high or low, its decisions will be based upon Cabrera A. cost of capital considerations. C. operating leverage considerations. B. financial leverage considerations. D. risk-profitability trade-off considerations. 71. When managing cash and short-term investments, a corporate treasurer is primarily concerned with A. Liquidity & safety B. Maximizing taxes. C. Maximizing the rate of return. D. Investing in Treasury bonds since they have no default mix. CMA 1295 72. The most important considerations with respect to short-term investments are A. Growth and value C. Return and value B. Return and risk D. Risk and liquidity

Wiley 2012

73. Investment instruments used to invest temporarily idle cash balances should have the following characteristics: Page 6 of 37

MANAGEMENT ADVISORY SERVICES

A. B. C. D.

low default risk, low marketability, and a short term to maturity. high expected return, readily marketable, and no maturity date. low default risk, readily marketable, and a short term to maturity. high expected return, low marketability, and a short term to maturity.

Working Capital Management

Cabrera

74. The primary factor to consider when evaluating any investment alternative as a use for shortterm excessive cash is the a. yield-to-maturity c. maturity date b. tax payable on the return d. safety of principal Cabrera 75. In the process of investing of surplus cash, the term “riding the yield curve” refers to A. purchasing only the longest maturities for given rates of return. B. adherence to the liquidity-preference theory of securities investments. C. swapping different maturities for similar quality debt securities in order to obtain higher yields. D. diversifying a securities portfolio so that the firm has an equal balance of long-term versus short-term securities. Cabrera 76. Which one of the following investment alternatives is commonly used to hold cash earmarked to meet a firm’s short term periodic variation in cash needs? A. Treasury Bills B. Treasury stock. C. High grade, corporate debentures. D. High grade, corporate mortgage bonds. Cabrera 77. In smaller business where the management of cash is but one of numerous functions performed by the treasurer, various cost incentives and diversification arguments suggest that surplus cash should be invested in A. banker’s acceptances. C. corporate bonds. B. commercial paper. D. money market mutual funds. Cabrera 78. Which of the following investments generally pay the highest return? A. Commercial paper C. Treasury bills B. Money market accounts D. Treasury notes

Wiley 2012

79. Short-term securities issued by the Federal Housing Administration are known as A. Agency securities C. Commercial paper B. Banker’s acceptance D. Repurchase agreements CMA 0689

80. Which one of the following is not a characteristic of a negotiable certificate of deposit? Negotiable certificates of deposit A. Have a secondary market for investors. B. Are regulated by the Banko Sentral ng Pilipinas C. Are usually sold in denominations of a minimum of P100,000 CMA 0691 D. Have yields considerably greater than bankers’ acceptances and commercial paper 81. All of the following are alternative marketable securities suitable for investment except A. RP Treasury Bills C. Commercial paper B. Eurodollars D. Convertible bonds CMA 0694* 82. One of the responsibilities of a financial manager is to make efficient use of idle cash for short periods of time. Which one of the following would not qualify as a satisfactory investment for idle cash? A. Bangko Sentral Treasury Bills C. Negotiable certificates of deposit. B. Common stocks of Manila Corporation D. Prime commercial paper Cabrera 83. Which of the following investments is not likely to be a proper investment for temporary idle cash? A. Treasury bills B. Commercial paper C. Treasury bonds due within one year D. Initial public offering of an established profitable conglomerate RPCPA 0595 84. The term short selling is the A. Selling of a security that is not owned by the seller. B. Selling of a security that was purchased by borrowing money from a banker. C. Betting that stock will increase by a certain amount within a given period of time. D. Selling of all the share you own in a company in an anticipation that the price will decline dramatically. CMA 1294 85. The economic order quantity (EOQ) formula can be adapted in order for a firm to determine the optimal mix between cash and marketable securities. The EOQ model assumes all of the following except that A. Cash flow requirements are random. B. The total demand for cash is known with certainty. C. An opportunity cost is associated with holding cash, beginning with the first peso. D. The cost of a transaction is independent of the peso amount of the transaction and interest rates are constant over the short run. CMA 0689 Page 7 of 37

MANAGEMENT ADVISORY SERVICES

86. Which of the following is/are true in relation to the Baumol model of cash management? A. The optimal cash balance falls when intent rates rise. B. The optimal cash balance rises when interest rates rise. C. The optimal cash balance rises when brokerage fees rise. D. Both A and C are correct. AICPA* 87. If a corporation held a marketable equity security for one year, the total return on investment for this security would be A. the sum of cash dividends for the year divided by the purchase price B. the capital gain (loss) on the stock for the year divided by the purchase price. C. the earnings per share on the stock for the year divided by the purchase price. D. the sum of the cash dividends received plus any capital gain (loss) for the year divided by the purchase price. Cabrera Receivables Management 88. The primary objective in the management of accounts receivable is A. to realized no bad debts because of the opportunity cost involved. B. to provide the treasurer of the corporation with sufficient cash to pay the company’s bills on time. C. to coordinate the activities of manufacturing, marketing, and financing so that the corporation can maximize its profits. D. to achieve that combination of sales volume, bad debt experience, and receivables turnover that maximizes the profits of the corporation. CMA* 89. An objective accounts receivable management is to have both the optimal amounts of receivables outstanding and bad debts. This balance requires the trade-off between the benefit of more credit sales and A. the cost of sales. B. more bad debts. C. a high accounts receivable turnover. Roque, 2011 D. the cost of accounts receivables, such as collection, interest and cost of bad debts. 90. The average collection period for a firm measures the number of days A. Beyond a typical account becomes delinquent B. For a typical check to “clear” through the banking system. C. After a typical credit sale is made until the firm receives the payment. CMA 1295 D. Beyond the end of the credit period before a typical customer payment is received.

Working Capital Management

91. The average collection period for a firm measures the number of days after a typical credit sale is made until the firm receives the payment. It should be related to the firm’s credit terms. For example, a firm that allows 2/10, net 30 should have an average collection period of A. ten days. B. twenty days. C. thirty days. D. somewhere between ten days and thirty days. Roque, 2011 92. An increase in the firm’s collection period means A. the firm’s current ratio is increasing. B. the firm’s collection expenses have fallen. C. the firm’s receivables turnover ratio is increasing. D. the firm has become less efficient in the collection of its receivables.

Cabrera

93. Which of the following represents a firm’s average gross receivables balance? I. Average age in days of receivables × average daily sales II. Average daily sales × average collection period III. Annual credit sales ÷ accounts receivable turnover A. I only C. I and II only B. II only D. I, II, and III Roque, 2011 94. Which of the following represents a firm’s average gross receivables balance? I. Days sales in receivables x accounts receivable turnover. II. Average daily sales x average collection period. III. Net sales / average gross receivables. A. I only. C. I and II only. B. II only. D. II and III only.

CMA 1296

95. At any point in time, the level of accounts receivable on a corporate balance sheet is least affected by which one of the following factors? A. “Tight money.” B. Credit standards of the seller. C. Collection practices of the seller. D. Length of the company’s production process. Cabrera 96. The level of accounts receivable will most likely increase as: A. Cash sales increase and number of days sales B. Credit limits are expanded, credit sales increase, and credit terms remain the same C. Credit limits are expanded, cash sales increase, and aging of the receivables is improved Page 8 of 37

MANAGEMENT ADVISORY SERVICES

Working Capital Management

D. Cash sales increase, current receivables ratio to past due increases, credit limits remain the same. RPCPA 0594 97. Changing a firm’s credit terms from 2/20, net/60 to 2/10, net/30 will generally A. increase average collection period and increase sales. B. reduce the average collection period and reduce sales. C. increase the average collection period and reduce sales. D. reduce the average collection period and increase sales.

RPCPA*

98. In a set of comparative financial statements, you observed a gradual decline in the net to gross ratio, (i.e., between net sales and gross sales). This indicates that: A. Sales volume is decreasing. B. The discount period is being lengthened. C. There is stiffening in the grant of discounts to the customers. D. There is adherence to the collection policies of the company. Agamata 2013 99. An aging of accounts receivable measures the A. ability of the firm to meet short-term obligations. B. average length of time that receivables have been outstanding. C. percentage of sales which have been collected after a given time period. D. amount of receivables that have been outstanding for given lengths of time. 100. The goal of credit policy is to A. Maximize sales. B. Minimize bad debts losses. C. Minimize collection expenses. D. Extend credit to the point where marginal profits equal marginal costs.

Cabrera

RPCPA 0597

101. It is held that the level of accounts receivable that the firm has or holds reflects both the volume of a firm’s sales on account and a firm’s credit policies. Which one of the following items is not considered as part of the firm’s credit policies? A. The length of time for which credit is extended. B. The size of the quantity discount that will be offered. C. The minimum risk group to which credit should be extended. D. The extent (in terms of money) to which a firm will go to collect an account. RPCPA 1095 102. The one item listed below that would warrant the least amount of consideration in credit and collection policy decisions is the A. cash discount given. C. quality of accounts accepted.

B. credit period.

D. quantity discount given.

103. All but which of the following is considered in determining credit policy? A. Accounts payable deferral period C. Credit limits B. Collection efforts D. Credit standards 104. Which of the following describes a firm’s credit criteria? A. The diligence to collect slow-paying accounts. B. The percentage of discount allowed for early payment. C. The length of time a buyer is given to pay for purchase. D. The required financial strength of acceptable customers.

Cabrera

Bobadilla

Wiley 2012

105. The procedures followed by the firm for ensuring payment of its accounts receivables are called its A. Collection policy C. Discount policy B. Credit policy D. Payables policy Wiley 2012 106. Which of the following statements is most correct? If a company lowers its DSO, but no changes occur in sales or operating costs, then the company A. might well end up with a lower debt ratio. B. might well end up with a higher debt ratio. C. would probably end up with a higher ROE. D. total asset turnover ratio would probably decline. Bobadilla 107. Following are ways of accelerating collection of accounts receivables, except A. shorten credit terms. B. minimize negative float. C. age accounts receivables. D. offer special discounts to those who pay promptly

Roque, 2011

108. A company’s president requested the credit and collection manager to submit proposals on the company’s credit policy. The credit and collection manager submitted two proposals. In both proposals, sales, profits, and collection period will change although by different periods. Bad debts experience will remain the same despite the proposed changes. In making a decision on which proposal should be implemented, the president should consider the following factors, except A. the cost of short-term credit. B. the company’s current bad debts experience. C. the impact of the proposed changes on the current customers of the company. Page 9 of 37

MANAGEMENT ADVISORY SERVICES

Working Capital Management

D. the change in credit terms to be imposed by banks which provide short-term financing to the company. Roque, 2011 109. A change in credit policy accelerated the collection of accounts receivable. As a result, the company experienced the following, except A. a decrease in bad debts. B. a decrease in the receivables balance. C. an increase in the average collection period. D. an increase in discounts taken by customers. Roque, 2011 110. A change in a seller’s credit policy has caused the following:  Sales decreased  Discounts taken decreased  Investment in accounts receivable increased  The number of doubtful accounts increased Based on this information, we can say that A. Net profit has decreased B. Gross profit has increased C. The average collection period has increased D. The company increased the rate of discount offered

Roque, 2011

111. A strict credit and collection policy is in place in Star Company. As Finance Director you are asked to advise on the property of relaxing the credit standards in view of stiff competition in the market. Your advise will be favorable if: A. The competitor will do the same thing to prevent lost sales. B. The projected margin from increased sales will exceed the cost of the incremental receivables. C. The account receivable level is improving so the company can afford the carrying cost of receivables. D. There is a decrease in the distribution level of your product and a more aggressive stance is necessary to retain market share. RPCPA 0594 112. A change in credit policy has caused an increase in discounts taken, a decrease in the amount of bad debts, and a decrease in the investment in accounts receivable. Based upon this information, the company’s A. working capital has decreased. B. average collection period has decreased. C. percentage discount offered has decreased. D. accounts receivable turnover has decreased. CMA 1296

113. A change in credit policy has caused an increase in sales, an increase in discounts taken, a reduction in the investment in accounts receivables, and a reduction in the number of doubtful accounts. Based on this information we know that A. The net profit has increased. B. The bad debt percentage has increased. C. The average collection period has decreased. D. The size of the discount offered has decreased. CMA 1289 114. The credit and collection policy of Levy Company provides for the imposition of credit block when the credit line is exceeded and/or the account is past due. During the month, because of the campaign to achieve volume targets, the general manager has waived the credit block policy in a number of instances involving big volume accounts. The likely effect of this move is A. Increase in the level or receivables only. B. Deterioration of aging of receivables only. C. Decrease in collections during the month the move was done. D. Deterioration of aging of receivables and increase in the level of receivables. RPCPA 1094 115. If a firm had been extending trade credit on a 2/10, net/30 basis, what changes would be expected on the balance sheet of its customer if the firm went to a net cash 30 policy? A. Decreased in cash. B. Increased receivables. C. Decreased receivables. D. Increased payables and increased bank loan. RPCPA 0596 116. Which one of the following statements is most correct if a seller extends credit to a purchaser for a period of time longer than the purchaser’s operating cycle? The seller A. Has no need for a stated discount rate or credit period. B. Is, in effect financing more than just the purchaser’s inventory needs. C. Can be certain that the purchaser will be able to convert the inventory into cash before payment is due. D. Will have a lower of accounts receivable than those companies whose credit period is shorter than the purchaser’s operating cycle. CMA 1296 117. The sales manager of Ryan Company feels confident that, if the credit policy at Ryan’s were changed, sales would increase and, consequently, the company would utilize excess capacity. The two credit proposals being considered are as follows: Page 10 of 37

MANAGEMENT ADVISORY SERVICES

Proposal A Proposal B Increase in sales ₱500,000 ₱600, 000 Contribution margin 20% 20% Bad debts percentage 5% 5% Increase in operating profits ₱ 75, 000 ₱ 90, 000 Desired return on sales 15% 15% Currently, payment terms are net 30. The proposed payment terms for Proposal A and Proposal B are net 45 and net 90, respectively. An analysis to compare these two proposals for the change in credit policy would include all of the following factors except the A. Cost of funds for Ryan. B. Current bad debt experience. C. Bank loan covenants on days’ sales outstanding CMA 0697 D. Impact on the current customer base of extending terms to only certain customers. 118. Which one of the following represents methods for converting accounts receivable to cash? A. Factoring, pledging, and electronic funds transfers B. Trade discounts, collection agencies, and credit approval C. Cash discounts, electronic funds transfers, and credit approval D. Cash discounts, collection agencies, and electronic funds transfers Cabrera Inventory Management 119. Inventory management is the formulation and administration of plans and policies to efficiently and satisfactorily meet production and merchandising requirements and minimize costs relative to inventories. One of its objectives is to A. maximize sales. B. minimize production costs. C. maximize the units in inventory. D. maintain inventory at a level that best balances the estimates of actual savings, the cost of carrying additional inventory and the efficiency of inventory control. Roque, 2013

Working Capital Management

D. a greater need for inspection of goods as the goods arrive.

Cabrera

122. The goal of managing working capital, such as inventory, should be to minimize the A. opportunity cost of capital B. costs of carrying inventory C. amount of spoilage or pilferage D. aggregate of carrying and shortage costs Bobadilla 123. Which inventory costing system will result in a high inventory turnover ratio in a period of rising prices? A. FIFO C. Periodic B. LIFO D. Perpetual Roque, 2013 124. A company would be willing to have a low inventory turnover ratio if the: A. carrying cost of inventory is high C. inventory order costs is low B. cost of stock out is high D. lead time is short Roque, 2013 125. Which of the following will not affect the budgeting of order getting costs? A. Location of distribution warehouses C. Policies and action of competitors B. Market research and test D. Sales promotion policies CMA 1279 126. Order-filling costs, as opposed to order-getting costs, include all but which of the following items? A. Credit check of new customs B. Packing and shipping of sales order C. Mailing catalogs to current customer D. Collection of payments for sales order CMA 1279

120. Economic order quantity model and two-bin system are commonly used controls for a company's material function. Those controls primarily relate to what part of the cycle? A. Material requirements C. Production distribution B. Physical storage D. Raw materials acceptance CIA 0588

127. The control of order filling costs A. Can be accomplished through the use of flexible budget standards B. Is related to pricing decisions, sales promotions, and customer reaction C. Is not crucial because the costs are typically fixed and not subject to frequent changes D. Is not crucial because the costs order-filling routine is entrenched and external influences are minimal. CMA 1279

121. Companies that adopt just-in-time purchasing systems often experience A. an increase in carrying costs. B. fewer deliveries from suppliers. C. a reduction in the number of suppliers.

128. Inventory costs, in addition to the costs of the purchased items, have been traditionally classified as follows, except A. carrying costs. C. order-filling costs. B. order costs. D. stockout costs. Roque, 2013 Page 11 of 37

MANAGEMENT ADVISORY SERVICES

Working Capital Management

129. The ordering costs associated with inventory management include A. Shipping costs, obsolescence, setup costs, and capital invested B. Insurance costs, purchasing costs, shipping costs, and spoilage. C. Obsolescence, setup costs, quantity discounts lost, and storage costs. D. Purchasing costs, shipping costs, setup costs, and quantity discounts lost

A. Cost of capital invested in the inventory C. Insurance Cost B. Cost of obsolescence D. Shipping Costs

CMA 0687

130. Inventory management requires the firm to balance the on hand for operations with the investment in inventory. Two cost categories in inventory management are order costs and carrying costs. A. the order costs include insurance costs, shipping costs and obsolescence B. the carrying costs include handling costs, interest on capital invested, and obsolesce. C. the carrying costs include purchasing costs, shipping costs, quantity discounts lost and setup cost. D. the order costs include quantity discounts lost, handling costs and setup costs for a production run. Roque, 2013 131. The carrying cost associated with inventory management includes A. Insurance cost, shipping costs, storage costs, and obsolescence B. Purchasing cost, shipping costs, set up costs, and quantity discount lost C. Storage costs, handling costs, interest on capital invested and obsolescence CMA 0687 D. Obsolescence, set up costs, interest on capital invested, and purchasing order costs 132. The carrying cost pertaining to inventory includes A. Insurance costs, incoming freight cost and setup cost B. Insurance costs, incoming freight cost and storage cost C. Setup and opportunity cost of capital invested in inventory D. Storage cost and opportunity cost of capital invested in inventory

RPCPA 0595

133. An example of a carrying cost is A. Disruption of production schedules B. Handling costs

C. Obsolescence D. Quantity discounts lost

CMA 1294

134. An example of carrying cost is A. Disruption of production schedules B. Handling cost

C. Quantity Discount lost D. Spoilage

CMA 1294

135. Which one of the following would not be considered a carrying cost associated with inventory?

136. If one optimizes the inventory turnover ratio, which cost will not increase? A. carrying cost C. total reorder costs B. stockout cost D. unit reorder cost

CMA 0697

RPCPA 1090

137. In inventory management, a decrease in frequency of ordering will normally: A. have no effect on total carrying cost C. increase total carrying cost B. have no effect on total ordering cost D. increase total ordering cost Roque, 2013 138. You computed the economic order quantity of the main raw material of Moonlight Company at 10,000 units. However, the chief purchasing officer decided to order in quantities of 12,000 units. What is the probable effect of the decision on the company's annual purchase order cost compared with the amounts had the order been made at the economic order quantity. A. Higher purchase order cost and higher carrying cost. B. Higher purchase order cost and lower carrying cost C. Lower purchase order cost and higher carrying cost D. Lower purchase order cost and lower carrying cost AICPA* 139. In inventory management the problem of avoiding excessive investment in inventories and at the same time avoiding inventory shortages can be solved by applying a quantitative technique known as: A. High-low point method C. Payback analysis B. Economic order quantity model D. Probability Analysis RPCPA 0589 140. Which of the following is used in determining the economic order quantity (EOQ) A. Regression analysis C. Markov process B. Calculus D. Queuing Theory

CIA 0593

141. The purpose of the economic order quantity model is to A. Minimize the safety stock. B. Minimize the inventory quantities. C. Minimize the sum of the order costs and the holding costs. D. Minimize the sum of the demand costs and the backlog costs.

CIA 0594

142. The order size determined by the economic order quantity formula minimizes the annual inventory cost which is comprised of ordering cost and A. Carrying Cost C. Stock out cost Page 12 of 37

MANAGEMENT ADVISORY SERVICES

B. Safety stock cost

Working Capital Management

D. No answer

RPCPA 0588

143. In the Economic Order Quantity (EOQ) model, some of the underlying assumptions are: A. Constant demand, constant ordering cost, constant carrying cost, and unlimited inventory capacity. B. Limited production capacity, declining demand, constant ordering cost, constant carrying cost and unlimited inventory capacity. C. Increasing demand, limited production capacity, increasing ordering cost, increasing carrying cost, and limited inventory capacity. D. Unlimited production capacity, declining demand, decreasing ordering cost, decreasing carrying cost, and unlimited inventory capacity. RPCPA 0594 144. The economic order quantity (EOQ) formula assumes that A. Periodic demand for the good is known B. Costs of placing an order vary with quantity order C. Erratic usage rates are cushioned by safety stocks D. Purchase costs per unit differ because of quantity discounts 145. The Economic Order Quantity (EOQ) formula does not assume that A. usage is uniform B. demand is known C. the cost of inventory itself is constant D. the cost of placing an order is constant

AICPA 0591

Roque, 2011

146. A characteristics of the basic economic order quantity (EOQ) model is that is A. is relatively insensitive to error B. is used when product demand, lead-time, and order costs are uncertain C. should not be used when carrying cost are large in relation to procurement cost D. Should not be used in conjunction with computerized perpetual inventory system 1294 147. One of the elements included in the economic order quantity (EOQ) formula is A. Safety stock C. Selling price of item B. Yearly demand D. Lead time for delivery

CMA

CIA 0595

148. In computing the economic order quantity (EOQ) which of the following cost should be included? A. Capital cost B. expected Value analysis

C. purchasing staff's salaries. D. The shipping cost to deliver the products to customers

RPCPA 0597

149. In the EOQ model, the return on capital that is foregone when it is invested in inventory is a (an) A. carrying cost C. irrelevant cost B. exclusion in the EOQ computation D. order cost Roque, 2011 150. To evaluate the efficiency of purchase transactions, management decides to calculate the economic order quantity for a sample of the company’s products. To calculate the economic order quantity, management would need data for all of the following, except the A. purchase prices of the products C. volume of products in inventory B. the fixed cost of ordering products D. volume of product sales Bobadilla 151. Which of the following is not an element in the EOQ formula? A. periodic carrying cost per unit C. variable cost per order B. safety stock D. yearly demand

Roque, 2013

152. Which one of the following items is not directly reflected in the basic economic order quantity (EOQ) model? A. Interest in invested capital B. Public warehouse rental charges C. Setup costs of manufacturing runs D. Quantity discounts lost on inventory purchases CMA 1294 153. The result of the economic order quantity formula indicates the A. Annual quantity of inventory to be carried B. Annual usage of materials during the year C. Quantity of each individual order during the year D. Safety stock plus estimated inventory for the year

CMA 0691

154. Which of the following statements is correct for a firm that currently has total costs of carrying and ordering inventory that are 50% higher than total carrying costs? A. Per unit carrying costs are too high B. Current order size is less than optimal C. Current order size is greater than optimal D. The optimal order size is currently being used Bobadilla 155. Firms that maintain very low or no inventory levels have Page 13 of 37

MANAGEMENT ADVISORY SERVICES

A. higher carrying costs B. higher ordering costs

Working Capital Management

C. higher ordering and carrying costs D. lower ordering and carrying Bobadilla

costs

156. With regards to inventory management, an increase in the frequency of ordering will normally? A. Have no impact on ordering costs C. Reduce total carrying costs B. Have no impact on total carrying costs D. Reduce the total cost CIA 0590 157. A decrease in inventory order costs will A. Increase the reorder point B. Decrease the economic order quantity C. Decrease the carrying cost percentage D. Have no effect on the economic order quantity 158. The economic order quantity (EOQ) will rise following A. An increase in carrying costs B decrease in annual unit sales C. An increase in the per-unit purchased price of inventory D. An increase in the variable costs of placing and receiving an order

RPCPA 0596

CIA 1189

159. The Stewart Company uses the EOQ model for inventory management. A decrease in which one of the following variable would increase the EOQ? A. Annual sales C. Cost per order B. Carrying costs D. Safety stock level CMA 1295 160. A decrease in inventory order cost will A. Decrease the EOQ B. Decrease the holding cost percentage

C. Have no effect on the EOQ D. Increase the reorder point RPCPA 0596

161. An increase in inventory carrying cost will A. Increase the safety stock required B. Decrease the economic order quantity C. Have no effect on the economic order quantity D. Decrease the number of orders issued per year

CMA 0691

162. As a consequence of finding a more dependable supplier, Dee Company reduced its safety stock of raw materials by 80%. What is the effect of this safety stock reduction on Dee’s Economic order quantity?

A. 64% decrease B. 80% decrease

C. 20% increase D. No effect

Wiley 2012

163. The selling price of the product is relatively high and the purchase cost of the product is relatively low. In this situation A. The EOQ model will B. The EOQ of the product is affected by the selling price C. The selling price has nothing to do with the EOQ of the product. RPCPA 0594 D. Management must increase the price to cover the cost of carrying higher inventory. 164. The use of EOQ analysis in inventory management can be modified to improve the management of inventory by A. purchasing inventory only once a year in order to save on ordering costs. B. purchasing inventory on a monthly basis in order to save on carrying costs. C. eliminating semi-variable costs from any consideration in the EOQ analysis due to the difficulty of estimating those costs. D. employing a minimum safety stock level because delivery times and inventory usage levels do not conveniently match quantitative formulas. CMA* 165. Ardmore Industries is in the process of reviewing its inventory and production policies. The company often has an excess supply needed for the planned shortages of other products needed for planned productions runs. The method that Ardmore should use to establish its inventory policies regarding these products is A. Contribution margin analysis C. Linear programming B. Economic production quantity analysis D. Regression analysis CMA 0688 166. The Economic Order Quantity (EOQ) model can be used to establish inventory policy. In the case of a manufacturer, the EOQ is called the Economic Lot Size (ELS) or Economic Production Quantity (EPQ). Which of the following statements about the ELS is incorrect? A. In the ELS model, the production rate is deemed to be instantaneous. B. In the ELS model, the demand is assumed to occur at a constant rate over some period of time. C. The ELS model is used to maximize contribution margin or minimize costs given resource constraints. D. The objective of the ELS model is to minimize the sum of inventory carrying costs and the costs of production runs or setup costs. Roque, 2013 167. The simple economic production lot size model will only apply to situation in which the production Page 14 of 37

MANAGEMENT ADVISORY SERVICES

A. B. C. D.

Rate equals the demand rate Rate is less than the demand rate Rate is greater than the demand rate For the period covered equal the projected sales for the period

Working Capital Management

Statement 1 Statement 2 CMA 0688

168. Which of the following statements is false? A. A decrease in inventory order costs will decrease the EOQ. B. An increase in inventory carrying costs will decrease the EOQ. C. An increase in the variable cost of placing and receiving an order will increase the EOQ. D. The cost of inventory itself, as well as many quantity discount lost on inventory purchases, is directly reflected in the EOQ model. Roque, 2013 169. Traditionally, large manufacturers have believed that economies of scale gained through large production runs of like, or similar, products are the best way to keep production cost down and remain competitive. Select the most appropriate response whether this theory is valid. A. No, economics of scale can no longer be gained from long production runs B. No, production flexibility and diversity of products are needed to remain competitive. C. Yes, larger economies of scale continues to accrue from ever larger production runs D. Yes, lower-per-unit costs for standard products continue to guarantee a competitive advantage Agamata 2013 170. State whether the statements are true or false. Statement 1: The two main types of inventory cost relevant to inventory decision-making are carrying costs and ordering costs Statement 2: The optimal ordering quantity in the EOQ model occurs at the point where the sum of the carrying costs and ordering costs are minimized. Agamata 2013 A. B. C. D. Statement 1 True True False False Statement 2 True False True False 171. Indicate whether the following statements are true or false/ Statement 1: The cost of warehousing and storage, property taxes, insurance of inventory, losses from spoilage are examples of ordering costs. Statement 2: One of the relevant costs in inventory management is “carrying cost” which refers to the total effect of the failure of a company to service customers or conduct manufacturing operations smoothly because of goods, raw materials and/or suppliers are out of stock. Agamata 2013 A. B. C. D.

True True

True False

False True

False False

172. The optimal level of inventory would be affected by all of the following except the A. cost per unit of inventory B. current level of inventory C. usage rate of inventory per time period. D. cost of placing an order for merchandise

Cabrera

173. The amount of inventory that a company would tend to hold in stock would increase as the A. variability of sales decreases B. cost of carrying inventory decreases C. cost of running out of stock decreases D. sales level falls to a permanently lower level Bobadilla 174. Which of the following would tend to increase the holdings of inventory quantity in the future? A. Increased computer control C. Limited variety of products B. Increased rate of sales growth D. Standardization of products Cabrera 175. The size of safety stocks for inventory is important for most firms. Though several factors can be cited as contributing to the determination of the size of safety stock that a firm should carry, the issue can often be reduced to a single factor. Which one of the following statements best summarizes the factor that affects the level of safety stock that a firm will carry? A. The rapidity with which the inventory position will turn over. B. The level of production the firm’s bank is willing to finance. C. The amount of idle cash management believes it has to invest in safety stock. D. The level of uncertainty with respect to an out-of-stock condition that management is willing to accept. RPCPA* 176. Safety stocks are used to compensate for A. inventory obsolescence and sales returns. B. variations in inventory prices and lead times. C. variations in inventory usage rates and prices. D. variations in inventory usage rates and lead times

Cabrera

177. The level of safety stock in inventory management depends on all of the following except A. cost to reorder stock B. cost of running out of inventory Page 15 of 37

MANAGEMENT ADVISORY SERVICES

Working Capital Management

C. level of uncertainty of the sales forecast. D. level of customer satisfaction for back orders.

178. In inventory management, the safety stock will tend to increase if the A. Carrying cost increases C. Fixed order cost decreases CMA 1289 B. Cost of running out of stock decreases D. Variability of the lead time increases

184. When a specified level of stock is carried for an item in inventory, the average inventory level for that item A. Is not affected by the safety stock B. Increase by the amount of the safety stock C. Decrease by the amount of the safety stock D. Increase by the one-half the amount of the safety stock RPCPA 0596

179. The use of safety stock by a firm will A. have no effect on inventory costs B. increase inventory costs

185. The elapsed time between placing an order for inventory and receiving the order is A. Lead time C. Stocking time B. Reorder time D. Stockout time CMA 0688

180. When the level of safety stock is increased A. Lead time will increase B. Order costs will decrease C. Carrying costs will decrease D. The frequency of stockouts will decrease

C. reduce inventory costs D. none of the given choices

Cabrera

Bobadilla

186. To determine the reorder point, calculations normally include the A. average daily usage C. economic order quantity B. carrying cost D. ordering cost CMA 0688

181. A company stocks, maintains, and distributes inventory. The company decides to add to the safety stock and expedite delivery for several product lines on a trial basis. For the selected product lines, the company will experience A. A change in the service level B. A decrease in ordering, carrying, and delivery costs C. An increase in ordering, carrying, and delivery costs D. An increase in some costs but no change in the service level CIA 1190 182. The amount of inventory that a company would tend to hold in safety stock would increase as the A. variability of sales decreases. B. cost of carrying inventory decreases. C. cost of running out of stock decreases. D. sales level falls to a permanently low level. Cabrera 183. For a 300-day work year Kulasa Corp. consumes 420,000 units of an inventory item. The usual lead time for the inventory item is six days; however at times the lead time has gone as high as eight days. Kulasa now desires to adjust its safety stock policy. The likely effect on stockout costs and carrying costs, respectively, would be. A. Decrease and decrease C. Increase and decrease B. Decrease and increase. D. Increase and increase RPCPA 0597

Wiley 2012

187. For inventory management, ignoring safety stocks, which of the following is a valid computation of the reorder point? A. The EOQ B. The EOQ times the anticipated demand during lead time C. The square root of the anticipated demand during the lead time D. The anticipated demand per day during lead time times lead time in days AICPA 0576 188. Which changes in costs are most conducive to switching from a traditional inventory ordering system to a just-in-time ordering system? Bobadilla A. B. C. D. Cost per purchase order Increasing Increasing Decreasing Decreasing Inventory unit carrying cost Increasing Decreasing Increasing Decreasing 189. Pepper Company changed from a traditional manufacturing philosophy to a just-in-time technology. What are the expected effects of this change on Pepper’s inventory turnover and inventory as a percentage of total assets reported on Pepper’s balance sheet? Bobadilla A. B. C. D. Inventory turnover Increase Increase Decrease Decrease Inventory percentage Increase Decrease Increase Decrease 190. The cost of stock-out do not include A. Depreciation and obsolescence. B. Disruption of production schedules

C. Loss of customer goodwill D. Loss of sales

RPCPA 1095 Page 16 of 37

MANAGEMENT ADVISORY SERVICES

191. What are the three factors a manager should consider in controlling stockouts? A. Carrying costs, quality costs, and physical inventories B. Economic order quantity, annual demand, and quality costs C. Economic order quantity, production bottlenecks & safety stock D. Time needed for delivery, rate of inventory usage, & safety stock

Working Capital Management

A. Advances by owners B. Commercial bank loan CIA 1191

192. A company experiences both variable usage rates and variable lead times for its inventory items. The probability distributions for both usage and lead times are known. A technique the company could use for determining the optimal safety stocks levels for an inventory items is: A. Decision tree analysis C. Monte Carlo simulation B. Linear Programming D. Queuing theory CMA 1291 193. A manufacturing company dates invoices seasonally so that skis delivered in September will bear an invoice due date for the following February. As a result of using this method, the manufacturer’s inventory carrying costs are (lower; higher; constant) and the buyer is extended to a (shorter; longer) credit period than would otherwise be true A. Higher; longer C. Lower; longer B. Higher; shorter D. Lower; shorter CIA 1182 Short-term Financing 194. Common sources of short-term financing include A. issuing bonds C. stretching payables B. reducing inventory D. all of the given choices

Bobadilla

195. Which of the following is not a source of short-term credit? A. accruals C. deferred income B. common stock D. purchases on account

Roque, 2011

196. Which of the following financial instruments generally provides the largest source of shortterm credit for small firms? A. Commercial paper C. Mortgage bonds B. Installment loans D. Trade credit CMA 1295 197. Which one of the following provides a spontaneous source of financing for a firm? A. Accounts payable C. Debentures B. Accounts receivable D. Mortgage bonds Becker 2009 198. Given that each of the following short-term resources in available, which source of financing is likely to have the highest cost for a small business?

C. Factoring D. Trade credit

Cabrera

199. An organization would usually offer credit terms of 2/10, net 30 when A. the cost of capital approaches the prime rate. B. the organization can borrow funds at a rate less than the annual interest cost. C. the organization can borrow funds at a rate exceeding the annual interest cost. D. most competitors are offering the same terms, and the organization has a shortage of cash. CMA* 200. With credit terms of 3/8, n/30, what is the customer’s payment decision date? A. Three days after the invoice is received. B. Anytime during the period, 8th to the 30th. C. The 30th day is the primary decision date. D. The 8th day is the customer’s decision date. 201. Which one of the following statements about trade credit is correct? A. Subject to risk of buyer default B. A source of long-term financing to the seller C. Not an important source of financing for small firms D. Usually an inexpensive source of external financing

Bobadilla

CMA 1296

202. On cash discounts, all of the following statements do not apply except A. The cost of not taking a cash discount is always higher than the cost of a bank loan. B. The cost of not taking the discount is higher for terms of 2/10, net 60 than for 2/10, net 30. C. With trade terms of 2/15, net 60, if the discount is taken the buyer receives 45 days of free credit. D. If a firm buys ₱10,000,00 of goods on terms of 1/10, net 30 and pays within the discount period, the amount paid would be 9,000. RPCPA 0597 203. As a corporation grows from a small business into a larger, more profitable business, the corporation tends to rely A. More on trade credit because of its unlimited availability. B. More on cash purchases due to the cash flow generated through collection of accounts receivable. C. More on government secured loans which are available to the larger more profitable firms in the economy. D. More on bank credit and similar resources rather than trade credit because trade credit is Page 17 of 37

MANAGEMENT ADVISORY SERVICES

not adequate to meet the financing needs beyond certain levels of growth.

Working Capital Management

Cabrera

204. Short-term debt financing generally has the following three characteristics from the viewpoint of the borrower when compared with long-term debt financing: A. Less flexibility, lower total cost, and lower risk B. Greater flexibility, higher total cost, and lower risk C. Greater flexibility, lower total cost, and greater risk D. Greater flexibility, higher total cost, and greater risk Cabrera 205. The following forms of short-term borrowings are available to a firm: • Floating lien • Bankers’ acceptances • Factoring • Lines of credit • Revolving credit • Commercial paper • Chattel mortgages The forms of short term borrowing that are unsecured credit are: A. Factoring, chattel mortgage, bankers’ acceptances, and line of credit B. Floating lien, chattel mortgage, bankers’ acceptances, and line of credit C. Floating lien, revolving credit, chattel mortgages, and commercial paper D. Revolving credit, bankers’ acceptances, line of credit, and commercial paper CMA 1286 206. A small retail business would most likely finance its merchandise inventory with A. a chattel mortgage. C. a terminal warehouse receipt loan. B. a line-of-credit. D. commercial paper. Cabrera 207. Short-term interest rates are A. Usually lower than long-term rates B. Usually higher than long-term rates C. Not significantly related to long-term rates D. Lower than long-term rates during periods of high inflation only 208. The prime rate is the A. Effective cost of a commercial bank loan B. Size of the commitment fee on a commercial bank loan C. Rate at which a bank borrows from the Bangko Sentral ng Pilipinas D. Rate charged on business loans to borrowers with high credit ratings

CMA 0691

CMA 0688*

209. A minimum checking account balance that a firm must maintain with a commercial bank is a A. Compensating balance C. Speculative balance B. Precautionary balance D. Transaction balance CIA 1190

210. A compensating balance A. can be held in the form of a banker’s acceptance. B. may be required in lieu of a fee for bank services. C. earns interest at the same rate as a saving deposit. D. increases the effective rate of return on savings account.

Cabrera

211. A compensating balance A. Is the amount of prepaid interest on a loan B. Is used to compensate for possible losses on a marketable securities portfolio C. Is a level of inventory held to compensate for variations in usage rate and lead time D. Compensates a financial institution for services rendered by providing it with deposits of funds. CMA 0688* 212. The net effect of a compensating balance requirement on a loan from the viewpoint of the borrower is A. the compensating balance has no effect on financing costs. B. the compensating balance will seldom be used if the loan maturity is less than 5 years. C. the effective borrowing costs will be lower than if the compensating balance were not required. D. the effective borrowing costs will be higher than if the compensating balance were not required. Cabrera 213. The prime lending rate of commercial banks is an announced rate and is often understated from the viewpoint of even the most credit-worthy firms. Which one of the following requirements always results in a higher effective interest rate? A. A floating rate for the loan period. B. The absence of a charge for any unused portion in the line of credit. C. A covenant that restricts the issuance of any new unsecured bonds during the existence of the loan. D. The imposition of a compensating balance with an absolute minimum that cannot be met by current transaction balances. CMA 1280 214. Experience with compensating balances in corporate bank account reveals that A. the use of compensating balances tends to lower the cost of borrowing. B. the use of compensating balances has no effect on the cost of borrowing. C. Banks tend to be very strict and require that compensating balances be maintained exactly as agreed. D. banks tend to be flexible in administering compensating balances and, based on the Page 18 of 37

MANAGEMENT ADVISORY SERVICES

different business conditions, allow fluctuations from the required level. 215. Commercial paper A. Has a maturity date of greater than 1 year. B. Has an interest rate lower than Treasury bills. C. Ordinarily does not have an active secondary market. D. Is usually sold only through investment banking dealers.

Working Capital Management

Cabrera

CMA 0691

216. The principal advantage of using commercial paper as a short-term financing instrument is that it A. offers security, i.e., collateral, to the lender. B. is readily available to &most all companies. C. can be purchased without commission costs. D. is generally cheaper than a commercial bank loan. Cabrera 217. Commercial paper tends to be quite popular with large, profitable corporation because A. the market distribution for commercial paper is very narrow B. purchasers of the commercial paper typically use this type of investment on a longterm basis C. even though interest costs are higher than the interest on ordinary bank loans, the interest is tax deductible D. interest costs are lower than the interest on ordinary bank loans and compensating balances are not required of borrowers Cabrera 218. Which one of the following responses is not an advantage to a corporation that uses the commercial paper market for short – term financing? A. This market provides a broad distribution for borrowing. B. This market provides more funds at lower rates than other methods provide. C. There are no restrictions as to the type of corporation that can enter into the market. D. The borrower avoids the expense of maintaining a compensating balance with a commercial bank. CMA 0696 219. Which of the following statements concerning commercial paper is false? A. Commercial paper can be issued by virtually all firms. B. Commercial paper is generally written for terms less than 270 days. C. Commercial paper generally carries an interest rate below the prime rate. D. Commercial paper is sold to money market mutual funds, as well as to other financial institutions and non-financial corporations. Cabrera

220. The credit instrument known as banker’s acceptance A. Is a timed draft payable on a specified date and guaranteed by the bank B. Is a method of sales financing in which the bank retains title to the goods until the buyer has completed payment C. Calls for immediate payment upon delivery of the shipping documents to the bank’s customer and acceptance of goods by the bank. D. Involves an invoice being signed by the banker upon receipt of goods, after which both the banker and seller record the transaction on their respective goods. CIA 0596 221. Which of the following forms of short-term borrowing is a secured credit? A. Banker’s acceptance C. Commercial paper B. Chattel mortgage D. Line of credit 222. An example of secured short-term financing is: A. A line of credit C. A warehouse receipt B. A revolving credit line D. Commercial paper 223. Inventory financing can take the form of a A. blanker lien. B. trust receipt

C. warehouse receipt. D. all of the above

Roque, 2011

CIA 1191

Cabrera

224. In assessing the loan value of an inventory, a banker will normally be concerned about the portion of inventory that is work-in-process because A. WIP generally has the lowest marketability of the various types of inventories B. WIP inventory usually has the highest loan value of the different inventory types C. WIP represents lower investments by a corporation as opposed to other types of inventories D. WIP inventory is relatively easy to sell because it does not represent a raw material or a finished product RPCPA 0596 225. From the viewpoint of a borrower, a field warehouse arrangement is frequently considered superior to a terminal (public) warehouse arrangement because A. pledged inventory is not removed from the borrower’s property to another specific location. B. pledged inventory can be released to the borrower in cases of emergency without authorization from the lender. C. a warehouse person is required for a public warehouse arrangement, but no supervision is required for a field warehouse. D. insurance does not have to be carried on the inventory when a field warehousing Page 19 of 37

MANAGEMENT ADVISORY SERVICES

arrangement is employed making the cost much less.

Working Capital Management

Cabrera

226. Factoring is a credit arrangement A. which involves the outright sale of accounts receivable to a factor. B. which should be used only as a last resort when all other sources of financing fail. C. in which the factor is free to request new receivables for those accounts it deems uncollectible D. in which the cash advances from the factor is essentially a loan secured by the eventual collection of the receivables factored. Cabrera 227. A firm which finances through a factor A. maintains a compensating balance. B. uses inventory as collateral for a loan. C. sells approved accounts receivable without recourse. D. uses another company to endorse or guarantee a loan.

Cabrera

228. The principal difference between factoring and pledging receivables rests in the fact that in factoring A. the accounts receivable are pledged on a non-notification basis. B. the accounts receivable are sold outright to a financial institution. C. the accounts receivable are merely pledged as security for a loan D. the financial institution factoring the accounts reserves the right to substitute newer receivables for those accounts that appear difficult to collect Cabrera 229. The most appropriate tool for determining in what month a short-term bank loan can be repaid is A. The monthly cash budget B. The asset turnover for the month C. The earning as a percent of sales for the month D. The monthly Statement of Changes in Financial Position Cabrera 230. Which of the following is incorrect? A. Trade credit usually bears no interest, so it is costless. B. Pledging of receivables is an example of secured short-term credit C. When a firm purchases goods or services on credit from a supplier, it automatically obtains short-term financing. Roque, 2011 D. Accruals or accrued expenses is a form of spontaneous financing which represents liabilities for services that have been provided to the company but have not been paid for

Problems Working Capital 1 The working capital RED Company at December 31, 2012 was 10,000,000. Selected information for the year 2013 for RED Company is as follows: Working capital provided from operations P1,700,000 Capital expenditure 3,000,000 Proceeds from short-term borrowings 1,000,000 Proceeds from long-term borrowings 2,000,000 Payments on short-term borrowings 500,000 Payments on long-term borrowings 600,000 Proceeds from issuance of common stocks 1,400,000 Dividends paid on common stock 800,000 What is RED working capital at December 31, 2013? A. P10,700,000 C. P11,500,000 B. P11,200,000 D. P12,000,000 Agamata 2013 2.

During 2013, Mason Company's current assets increased by P120, current liabilities decreased by P50, and net working capital A. Decreased by P170 C. Increased by P70 B Did not change D. Increased by P170 CMA 1290

Liquidity Ratios 3. A firms current ratio is currently 1.70 to 1. Management knows it cannot violate a working capital restriction contained in its bond indenture. If the firm's current ratio falls below 1.40 to 1, technically it will have defaulted. If current liabilities are P200 million, the maximum new commercial paper that can be issued to finance inventory expansion is A. P80 million C. 280 million B. P150 million D. P370 million Gleim 4.

MFA Corporation has 100, 000 shares of stock outstanding. Below is part of MFC's Statement of Financial Position for the fiscal year. MFA Corporation Statement of Financial Position - Selected Items December 31, 2013 Cash Accounts receivable Inventory

P455, 000 900, 000 650, 000 Page 20 of 37

MANAGEMENT ADVISORY SERVICES

Prepaid assets 45, 000 Accrued liabilities 285, 000 Accounts payable 550, 000 Current portion, long term notes payable 65, 000 What is the amount MFA can pay in cash dividends per share and maintain a minimum current ratio of 2 to 1. Assume that all accounts other than cash remains unchanged. A. P2.05 C. P3.55 B. P2.50 D. P3.80 CMA 0697 Cash & Marketable Securities Management 5 If the average age of inventory is 60 days, the average age of the accounts payable is 30 days, and the average age of accounts receivables is 45 days, the number of days in the cash flow cycle is A. 75 days C. 105 days B. 90 days D. 135 days Gleim 6.

The following data are taken from the records of Apple Corporation for the year ended December 31, 200B: Net credit sales 576,000 Average materials inventory 8,000 Average finished goods inventory 12,000 Average accounts receivable 80,000 Average accounts payable 5,000 Net credit purchases 120,000 Raw materials used 96,000 Gross profit rate 25% What is the average number of days in the company’s operating cash conversion cycle? (use a 360-day year) A. 45 days C. 75 days B. 50 days D. 105 days Roque, 2011

Questions 7 & 8 are based on the following information. Bobadilla Samaritan Supplies, Inc. has P5 million in inventory and P2 million in accounts receivable. Its average daily sales are P100,000. The company has P1.5 million in accounts payable. Its average daily purchases are P50,000. 7.

What is the length of the company’s inventory conversion period? A. 40 days C. 90 days B. 50 days D. 120 days

Working Capital Management

8.

What is the length of the company’s cash conversion period? A. 20 days C. 40 days B. 30 days D. 50 days

9.

Casie Company turns out 200 calculators a day at a cost of P250 per calculator for materials and variable conversion cost. It takes the firm 18 days to convert raw materials into calculator. Casie’s usual credit terms extended to its customers is 30 days, and the firm generally pays its suppliers in 20 days. If the foregoing cycles are constant, what amount of working capital must Casie Company finance? A. P 900,000 C. P1,800,000 B. P1,400,000 D. P2,400,000 Bobadilla

10. Assume that each day a company writes and received checks totaling P10,000. If it takes 5 days for the checks to clear and be deducted from the company’s account, and only 4 days for the deposits to clear, what is the float? A. P(10,000) C. P10,000 B. P0 D. P50,000 CMA 0694 11. Annabelle Corporation is engaged in a multi-level marketing business that presently requires all sales agents to mail checks to its Manila office. An average of three days is required for mailed checks to be received, one day for Annabelle Corporation to process them and three days to clear through its banks. The company's treasurer proposed a change in the system, where the checks will no longer be mailed to Manila office. Instead, checks collected will be deposited on-line in any branch of the company's depositary bank, and the deposit slips, as well as the other pertinent documents will be sent by fax or e-mail to the Manila office on the same day. The original deposit slips and other documents will be submitted by the sales agents to Manila when they attend the Sales Agents' Monthly Meeting. The new system will eliminate the mailing float and the processing time. Annabelle Corporation has an average daily collection of P50,000. If the new system is implemented, Annabelle Corporation's average cash balance will increase by A. P50,000. C. P200,000. B. P150,000 D. P350,000. Roque, 2011 12. A firm has a daily cash receipts of P100,00 and collection time of 2 days. A bank has offered to reduce the collection time on the firm’s deposits by 2 days for a monthly fee of P500. If Page 21 of 37

MANAGEMENT ADVISORY SERVICES

money market rates are expected to average 6% during the year, the net annual benefit (loss) from having this service is A. P 0 C. P6,000 B. P3,000 D. P12,000 CMA 0696 13. What are the expected annual savings from a lock-box system that collects 150 checks per day averaging P500 each, and reduces mailing and processing times by 2.5 and 1.5 days respectively, if the annual interest rate is 7%? A. P 5,250 C. P 21,000 B. P 13,125 D. P300,000 Bobadilla 14. Average daily cash outflows are P3 million for Evans, Inc, A new cash management system can add two days to the disbursement schedule. Assuming Evans earns 10 percent on excess funds, how much should the firm be willing to pay per year for this cash management system? A. P600,000. C. P 3,000,000 B. P1,500,000. D. P6,000,000 Cabrera 15. RMN is a retail mail order firm that currently uses a central collection system that requires all checks to be sent to its flotation headquarters. An average of 6 days is required for mailed checks to be received, 3 days for RMN to process them, and 2 days for the checks to clear through its bank. A proposed lockbox system would reduce the mailing processing time to 2 days and the check clearing time for 1 day. RMN has an average daily collection of P150,000. If RMN adopts the lockbox system, its average cash balance will increase by A. P450,000 C. P750,000 B. P600,000 D. P1,200,00 Gleim 2013 16. A firm has a daily cash receipts of P300,000. A bank has offered to provide a lockbox service that will reduce the collection time by 3 days. The bank requires a monthly fee of P2,000 for providing this service. If monthly market rates are expected to average 6% during the year, the additional income (loss) of using the lockbox system is A. P(24,000) C. P30,000 B. P12,000 D. P54,000 Gleim 2013 17. Troy Toy is a retailer operating in several cities. The individual store managers deposit daily collections at a local bank in a non-interest bearing checking account. Twice per week, the local bank issued a depository transfer check (DTC) to the central bank at headquarters. The controller of the company is considering using a wire transfer instead. The additional cost of each transfer would be P25; collections would be accelerated by 2 days, and the annual

Working Capital Management

interest rate paid by the central bank is 7.2% (0.02% per day). At what amount of pesos transferred would it be economically feasible to use a wire transfer instead of the DTC? Assume a 350-day year. A. P 125,000 or above. B. Any amount greater than P 173. C. Any amount greater than P 62,500. D. It would never be economically feasible. CMA 1294 18. Globe Products have received proposals from several banks to establish a lock box system to speed up receipts. Globe receives an average of 700 checks per days averaging P1,800 each, and its cost of short term funds is 7% per year. Assuming that all proposals will produce equivalent processing results and using a 360-day year, which one of the following proposals is optimal for Globe? A. P0.50 per check B. A flat fee of P125,000 C. A fee of 0.03% of the amount collected D. A compensating balance of P1,750,000 CMA 0697 19. Majority of Aning Company's customers are farmers from remote rural areas. Farmers bank has offered to provide Aning Company a lockbox system at a fixed fee of P300 per month and a variable fee of P2 for each payment processed by the bank. Aning Company receives 30 payments per day, averaging P5,000 per payment. With the lockbox system, the company's collection float will decrease by 3 days. Money market securities earn 5% per annum. Should Aning Company accept Farmer Bank's offer to provide a lockbox system? (Use 360 days in a year) A. Yes, because it would earn additional income of P22,500 per year. B. Yes, because it would earn net benefit of P2,700 from the lockbox system. C. No, because the lockbox system would require the company to spend P25,000 per year. D. No, because the cost of the lockbox system is P2,700 more than the expected return on money market placements. Roque, 2011 20. Foster, Inc. is considering implementing a lockbox collection system at a cost of P80,000 per year. Annual sales are P90,000, and the lock-box system will reduce collection by 3 days. If Foster can invest funds at 8 percent, should it use the lock-box system? Assume a 360-day year. A. No, producing a loss of P20,000 per year. B. No, producing a loss of P60,000 per year. C. Yes, producing savings of P60,000 per year. Page 22 of 37

MANAGEMENT ADVISORY SERVICES

D. Yes, producing savings of P140,000 per year.

Working Capital Management

Cabrera

21. Butit is a newly established janitorial firm, and the owner is deciding what type of checking account to open. Butit is planning to keep a P500 minimum balance in the account for emergencies and plans to write roughly 80 checks per month. The bank charges P10 per month plus P0.10 per check charge for a standard business checking account with no minimum balance. Butit also has the option of a premium business balance that requires a P2,500 minimum balance but has no monthly fees or per check charges. If Butit’s cost of funds is 10%, which account should Butit choose? A. Premium account, because the savings is P16 per year B. Premium account, because the savings is P34 per year C. Standard account, because the savings is P16 per year D. Standard account, because the savings is P34 per year CMA 0697 22. Assuming a 360-day year, the current price of a P100 Treasury bill due in 180 days on a 6% discount basis is A. P93.00 C. P97.00 B. P94.00 D. P100.00 CMA 0694 23. Mahogany Company has a total annual cash requirement of P6,075,000 which are to be paid uniformly. Mahogany has the opportunity to invest the money at 8 percent per annum. The company spends, on the average, P45 for every cash conversion to cash and vice versa. What is the optimal conversion size for cash? A. P41,335 C. P60,000 B. P58,457 D. P82,670 Bobadilla 24. Hyperbole Corporation estimates its total annual cash disbursements of P3,251,250 which are to be paid uniformly. Hyperbole has the opportunity to invest the money at 9% per annum. The company spends, on the average, P25 for every cash conversion to marketable securities and vice versa. What is the opportunity cost of keeping cash in the bank account? A. P 188.55 C. P3,825.00 B. P1,912.50 D. P4,190.00 Bobadilla 25. A company uses the following formula in determining the optimal level of cash C= √2𝑏𝑡⁄𝐼 if: b = fixed cost per transaction I = interest rate on marketable securities t = total demand for cash over a period of time This formula is a modification of the economic order quantity (EOQ) formula used for

inventory management. Assume that the fixed cost of selling marketable securities is P10 per transaction and the interest rate on marketable securities is 6% per year. The company estimates that it will make cash payments of P12,000 over the one month period. What is the average cash balance (rounded to the nearest peso)? A. P 1,000 C. P 3,464 B. P 2,000 D. P 6,928 CMA 0696 Questions 26 and 27 are based on the following information: Roque 2011 Ben Corporation uses the Baumol Cash Management Model to determine its optimal cash balance. For the coming year, the expected cash disbursements total P432,000. The interest rate on marketable securities is 5% per annum. The fixed cost of selling marketable securities is P8 per transaction. 26. Using the Baumol Cash Management Model, the company's optimal cash balance is A. P1,175.76. C. P11,757.55. B. P5,878.78. D. P142,000.00. 27. Using the Baumol Cash Management Model, the average cash balance is A. P1,175.76. C. P11,757.55. B. P5,878.78. D. P142,000.00. Receivable Management 28. Piston, Inc. has an inventory conversion period of 60 days, a receivable conversion period of 35 days, and a payment cycle to 26 days. If its sales for the period just ended amounted to P972,000, what is investment in accounts receivables? (Assume 360 days in a year) A. P72,450 C. P85,200 B. P79,600 D. P94,500 RPCPA 0595 29. A firm averages P4,000 in sales per day and is paid, on average, within 30 days of the sale. After they receive their invoice, 55% of the customers pay by check, while the remaining 45% pay by credit card. Approximately how much would be the company show in accounts receivable on its balance sheet on any given data? A. P4,000 C. P54,000 B. P48,000 D. P120,000 CMA 1294 30. Jackson Distributors sells to retail stores on credit terms of 2/10, net 30. Daily sales average 150 units at a price of P300 each. Assuming that all sales are on credit and 60% of customers take the discount and pay on day 10, while the rest of the customers pay on day 30, the amount of Jackson’s accounts receivable is Page 23 of 37

MANAGEMENT ADVISORY SERVICES

A. P810,000 B. P900,00

Working Capital Management

C. P990,000 D. P1,350,000

CMA 1295

31. Mercado offers its customers credit terms of 5/10, net 20. One-third of the customers take the cash discount and the remaining customers pay on day 20. On average, 20 units are sold per day, priced at P10,000 each. The rate of sales is uniform throughout the year. Using a 360-day year, the company has days’ sales outstanding in accounts receivable, to the nearest full day, of A. 13 days C. 17 days B. 15 days D. 20 days CIA 1195 32. Clauson Inc. Grants credit terms of 1/15, net 30 and projects gross sales for next year of P2,000,000. The credit manager estimates that 40% of their customers pay on the discount date, 40% on the net due date, and 20% pay 15 days after the net due date. Assuming uniform sales and a 360-day year, what is the projected days’ sales outstanding (rounded to the nearest whole day)? A. 20 days. C. 27 days. B. 24 days. D. 30 days. CMA 0697 33. Simba Corporation whose gross sales amounted to P1,200,000 sold on terms of 3/10, net 30. The collections manager estimated that 30% of the customers pay on the tenth day and take discounts; 40% on the thirtieth days; and the remaining 30% pay, on average, 40 days after the purchase. If management would toughen on its collection policy and require that all nondiscount customers pay on the thirtieth day, how much would be the receivables balance? A. Zero C. P80,000 B. P50,000 D. P70,000 RPCPA 0595 34. The Liberal Sales Company’s budgeted sales for the coming year are P30 million of which 80% are expected to be made on credit. The company wants to change its credit terms from n/30 to 2/10, n/30. If the new credit terms are adopted, the company estimates that cash discounts would be taken on 40% of the credit sales and the uncollectible amount would be unchanged. The adoption of the new credit terms would result in expected discount availed of in the coming year of A. P192,000 C. P480,000 B. P288,000 D. P600,000 RPCPA 0596 35. Palm Company’s budgeted sales for the coming year are P40,500,000 of which 80% are expected to be credit sales at terms of n/30. Palm estimates that a proposed relaxation of credit standards will increase credit sales by 20% and increase the average collection period

from 30 days to 40 days. Based on a 360-day year, the proposed relaxation of credit to standards will result in an expected increase in the average accounts receivable balance of A. P 540,000 C. P1,620,000 B. P 900,000 D. P2,700,000 CMA 1292 36. Green Company’s budgeted sales for the coming year are P96 million, of which 80% are expected to be credit sales at terms of n/30. The company estimates that a proposed relaxation of credit standards would increase credit sales by 30% and increase the average collection period from 30 days to 45 days. Based on a 360-day year, the proposed relaxation of credit standards would result to an increase in accounts receivable balance of A. P1,920,000 C. P6,080,000 B. P2,880,000 D. P6,880,000 RPCPA 0595 37. Prest Corporation plans to tighten its credit policy. Below is the summary of changes Old New Average number of days collection 75 50 Ratio of credit to total sales 70% 60% Projected sales for the coming year is P100 million and it was estimated that the new policy will be a 5% less if the new policy is implemented. Assuming a 360-day year, what is the effect of the new policy on accounts receivable? A. No change. C. Decrease of P6,666,667. B. Decrease of P5 million. D. Decrease of P13 million. RPCPA 0596 38. Currently, La Carlota Company has annual sales of P2,500,000. Its average collection period is 45 days, and bad debts are 3 percent of sales. The credit and collection manager is considering instituting a stricter collection policy, whereby bad debts would be reduced to 1.5 percent of total sales, and the average collection period would fall to 30 days. However, sales would also fall by an estimated P300,000 annually. Variable costs are 75 percent of sales and the cost of carrying receivables is 10 percent. Assume a tax rate of 40 percent and 360 days per year. What would be the decrease in investment in receivables if the change were made? A. P 9,688 C. P 96,875 B. P 12,988 D. P129,975 Bobadilla 39. Mr. S Mart assumed the presidency of Riches Corp. He instituted new policies with respect to credit policy. Below is a summary of relevant information: Credit policy Sales P1,800,000 P1,980,000 Average collection period 30 days 36 days Page 24 of 37

MANAGEMENT ADVISORY SERVICES

The company requires a rate of return of 10% and a variable cost ratio of 60%. Using a 360day year, the pre-tax cost of carrying the additional investment in receivables under the new policy would be A. P2,880 C. P4,080 B. P3,000 D. P4,800 RPCPA 1096 40. Best Computers believes that its collection costs could be reduced through modification of collection procedures. This action is expected to result in a lengthening of the average collection period from 28 days to 34 days; however, there will be no change in the uncollectible accounts. The company’s budgeIf a firm purchases ted credit sales for the coming year are ₱27,000,000, and short-term interest rates are expected to average 8%. To make the changes in collection procedures cost beneficial, the minimum savings in collection costs (using a 360-day year) for the year would have to be A. ₱30,000 C. ₱180,000. B. ₱36,000 D. ₱360,000 CMA 1292 41. A company with P4.8 million in credit sales per year plans to relax credit standards, projecting that this will increase credit sales by P720,000. The company’s average collection period for new customers is expected to be 75 days, and the payment behavior of the existing customers is not expected to change. Variable cost are 80% of sales. The firm’s opportunity cost is 20% before taxes. Assuming a 360-day year, what is the company’s benefit(loss) on the planned changed in credit terms? A. P0 C. P120,000 B. P28,800 D. P144,000 Agamata 2013 42. May Corporation’s average annual sales is ₱6,400,000, 10% of which is cash sales. The variable cost ratio is 60%. Starting next year, May Corporation will relax its credit standards. The relaxation in credit standards is expected to cause the following changes:  Total credit sales will increase by 20%.  The collection period for incremental sales is 60 days. (The payment behavior of the existing customers will not change.)  The variable cost ratio, even for the incremental sales, will be the same as in the past. The cost of borrowing is estimated at 25% per year. The company uses 360 days in a year in all its computations. What is May Corporation’s expected benefit (loss) from the planned relaxation in the credit policy? A. (₱ 27,520) C. ₱ 460,800 B. ₱ 432,000 D. ₱ 1,152,000 Roque, 2011

Working Capital Management

43. Sisa Corporation has the following data: Selling price per unit ₱ 70 Variable cost per unit ₱ 45 Annual credit sales – units 50,400 Collection period 30 days Rate of return 20% Sisa Corporation is considering easing its credit standards. If it does, sales will increase by 25%; collection period will increase to 45 days; bad debts losses are anticipated to be 4% of the incremental sales; and collection costs will increase by ₱31,645. If the proposed relaxation in credit standard is implemented, the net benefit (loss) for Sisa Corporation is A. (₱ 100,000) C. ₱ 215,000 B. (₱ 33,075) D. ₱ 315,000 Roque, 2011 44. The Sales Director of Go Company suggests that certain credit terms be modified. He estimates the following effects:  Sales will increase by at least 20%  Accounts receivable turnover will be reduced to 8 times from the present turnover of 10 times  Bad debts, now at 1% of sales will increase to 1.5%. Sales before the proposed changes is at P900,000. Variable cost ratio is 55% and desired rate of return is 20%. Fixed expenses amount to P150,000 Should the company allow the revision of its credit terms? A. Yes, because income will increase by P64,000. P68,850 B. Yes, because losses will be reduced by P78,800. C. No, because income will be reduced by P13,000. D. No, because losses will be increased by P28,000. RPCPA 0594 45. Crest Company has the opportunity to increase annual sales by P1 million by selling to new riskier customers. It has been estimated that uncollectible expenses would be 15% and collection costs, 5%. The manufacturing and selling costs are 70% of sales and corporate tax is 35%. If they pursue this opportunity, the after-tax profit will: A. Increase by P35,000. C. Increase by P97,500. B. Increase by P65,000. D. Remain the same. RPCPA 0596 46. Wasting Resource Company has annual credit sales of P4 million. Its average collection period is 40 days, and bad debts are 5% of sales. The credit and collection manager is considering instituting a stricter collection policy, whereby bad debts would be reduced to 2% of total sales, and the average collection period would fall to 30 days. However, sales would Page 25 of 37

MANAGEMENT ADVISORY SERVICES

also fall by an estimated P500,000 annually. Variable cost is 60% of sales and the cost of carrying receivables is 12%. Assuming a tax rate of 35% and 360 days a year, the incremental change in the profitability of the company if stricter policy would be implemented would be A. A reduction in net income by P35,400. B. A reduction in net income by P38,350. C. A reduction in net income by P70,000. D. Zero as the positive and negative effects offset each other. RPCPA 0597 Questions 47 & 48 are based on the following information. Roque Che-Che Corporation is planning to change its credit policy. The proposed change is expected to:  Shorten the collection period from 50 days to 30 days.  Increase the ratio of cash sales to total sales from 20% to 30%  Decrease total sales by 10%. 47. If projected sales for the coming year is ₱40M, what is the peso impact on the average accounts receivable balance of the proposed change in the credit policy? (Use 360 days in a year.) A. ₱ 18,889 decrease C. ₱ 2,344,444 decrease B. ₱ 2,100,000 decrease D. ₱ 6,800,000 decrease 48. What is the impact of the proposed credit policy on the company’s accounts receivable turnover? A. Decrease by 7.2 C. Increase by 4.8 B. Decrease by 20 days D. Increase to 4.8 times Questions 49 & 50 are based on the following information Roque 2011 Elaine Corporation is planning to introduce changes in its collection procedures. The new procedures are expected to make its collection period longer by ten days, although there will be a change in bad debts. For the coming year, Elaine Corporation’s budgeted sales is ₱32,400,000 or ₱90,000 per day. Short-term interest can be expected to average at 9% per annum. 49. As a result of the changes in collection procedures, Elaine Corporation’s average accounts receivable balance will increase (decrease) by A. (₱900,000) C. ₱900,000 B. ₱90,000 D. ₱32,400,000

Working Capital Management

collection costs for the coming year should be A. ₱8,100. C. ₱90,000. B. ₱81,000. D. ₱900,000. Questions 51 and 52 are based on the following information: RPCPA 1095 Slippers Mart has sales of P3 million. Its credit period and average collection period are both 30 days and 1% of its sales end up as bad debts. The general manager intends to extend the credit period to 45 days which will increase sales by P300,000. However, bad debts losses on the incremental sales would be 3%. Costs of products and related expenses amount to 40% exclusive of the cost carrying receivables of 15% and bad debts expenses. 51. Assuming 360 days a year, the changes in policy would result to incremental investments in receivables of A. P9,750 C. P65,000 B. P24,704 D. P162,500 52. The change in the credit policy would result to increase (decrease) in incremental profit of A. P106,000 C. P171,000 B. P161,250 D. P177,750 Question Nos. 53 through 55 are based on the following data: Bobadilla Sonata Company is considering changing its credit terms from 2/15, net 30 to 3/10, net 30 in order to speed collections. At present, 40 percent of Sonata Company‘s customers take the 2 percent discount. Under the new term, discount customers are expected to rise to 50 percent. Regardless of the credit terms, half of the customers who do not take the discount are expected to pay on time, whereas the remainder will pay 10 days late. The change does not involve a relaxation of credit standards; therefore bad debt losses are not expected to rise above their present 2 percent level. However, the more generous cash discount terms are expected to increase sales from P2 million to P2.6 million per year. Sonata Company’s variable cost ratio is 75 percent, the interest rate on funds invested in accounts receivable is 9 percent, and the firm’s income tax rate is 40 percent. 53. What are the days sales outstanding (DSO) before and after the change of credit policy? A. 21.5 days and 22.5 days respectively C. 22.5 days and 27.0 days, respectively B. 22.5 days and 21.5 days, respectively D. 27.0 days and 22.5 days, respectively 54. The incremental carrying cost on receivable is A. P 643.75 C. P6,667.00 B. P 843.75 D. P8,889.00

50. To make the changes in collection procedures cost beneficial, the minimum savings in Page 26 of 37

MANAGEMENT ADVISORY SERVICES

55. The incremental after tax profit from the change in credit terms is A. P57,615 C. P65,640 B. P60,615 D. P68,493 Inventory Management 56. Julia Company has P5 million of average inventory and sales of P30 million. Using a 365-day year, calculate the firm’s inventory conversion period. A. 30.25 days C. 60.83 days B. 45.00 days D. 72.44 days Bobadilla 57. What is the inventory period for a firm with an annual cost of goods sold of P8 million, P1.5 million in average inventory, and a cash conversion cycle of 75 days? A. 6.56 days C. 52.60 days B. 18.75 days D. 67.50 days Bobadilla 58. Southern Company’s budgeted sales and budgeted cost of sales for the coming year are P144,000,000 and P90,000,000 respectively. Short-term interest rates are expected to average 10%. If Southern can increase inventory turnover from its present level of nine times per year to a level of 12 times per year, its cost savings in the coming year would be expected to be A. P250,000 C. P450,000 B. P400,000 D. P600,000 AICPA* 59. A & B Company’s financial plan for next year shows sales of P72 million and cost of sales of P45 million. It expects short term interest rates to average 10% for the coming year. It aims to increase inventory turnover from the present level of 9 times to 12 times next year. If its plans and objectives would be carried out, how much is the cost savings for the coming year? A. P125,000 C. P375,000 B. P300,000 D. P500,000 RPCPA 1096 60. Diesel Fashion estimates that 90,000 zippers will be needed in the manufacture of high selling products for the coming year. Its supplier quoted a price of P25 per zipper. Diesel planned to purchase 7,500 units per month but its supplier could not guarantee this delivery schedule. In order to ensure availability of these zippers, Diesel is considering the purchase of all these 90,000 units on January 1. Assuming Diesel can invest cash at 12%, the company’s opportunity cost of purchasing the 90,000 units at the beginning of the year is A. P123,750 C. P135,000 B. P127,500 D. P264,000 Bobadilla

Working Capital Management

61. Jeff company sells 20,000 radios evenly throughout the year. The cost of carrying one unit of inventory for one year is P8, and the purchase order cost per order is P32. What is the economic order quantity? A. 200 C. 400 B. 283 D. 625 RPCPA 0598 62. The following data relate to inventories for a given year of Cloud Company: EOQ 7,500 units Cost to place one purchase order P75 Total cost to place purchase orders for the year P15,000 Cost to carry one unit for one year P6 The estimated annual usage in units would be: A. 1,250,000 C. 2,250,000 B. 2,000,000 D. 5,625,000 RPCPA 1088 63. The basic EOQ model equals the square root of the (1) product of twice the demand times the cost per order, (2) divided by the periodic carrying cost per unit. If the annual demand increases by 44%, the EOQ will increase (decrease) by A. 6.63% C. 12% B. 9.38% D. 20% CIA 1195* 64. Narra Company is considering a switch to level production. Cost efficiencies will occur under level production and after tax cost would decline by P70,000 but inventory would increase from P1,000,000 to P1,800,000. Narra would have to finance the extra inventory at a cost of 10.5 percent. What is the maximum interest rate that makes level production feasible? A. 5.83 percent C. 8.75 percent B. 7.00 percent D. 10.00 percent Bobadilla 65. One of the products Nature Health Products sells is a magnetic back support. The ordering cost related to this product is P12.50 per order. The cost of carrying one item of inventory for one year is P16.00. The business sells 40,000 of this type of product evenly throughout the year. How much is the total ordering costs per year and total carrying cost per year at the EOQ? RPCPA 1092 A. B. C. D. Ordering cost P1,562.50 P1,562.50 P2,000 P4,000 Carrying cost P1,562.50 P2,560.50 P2,000 P4,000 66. Durable Furniture Company uses about 200,000 yards of a particular fabric each year. The Page 27 of 37

MANAGEMENT ADVISORY SERVICES

fabric costs P25 per yard. The current policy is to order the fabric four times a year. Incremental ordering costs are about P200 per order, and incremental carrying costs are about P0.75 per yard, much of which represents the opportunity cost of the funds tied up in inventory. How much total annual costs are associated with the current inventory policy? A. P18,750 C. P38,300 B. P19,550 D. P62,500 Bobadilla 67. BIBO Company is a distributor of videotapes. Pirate Mart is a local retail outlet which sells blank and recorded videos. Pirate Mart purchases tapes from BIBO Company at P300.00 per tape; tapes are shipped in packages of 20. BIBO Company pays all incoming freight, and Pirate Mart does not inspect the tapes due to BIBO Company's reputation for high quality. Annual demand is 104,000 tapes at a rate of 4,000 tapes per week. Pirate Mart earns 20% on its cash investments. The purchase-order lead time is two weeks. The following cost data are available: Relevant ordering costs per purchase order P90.50 Carrying costs per package per year: Relevant insurance, materials handling, breakage, etc., per year P 4.50 What is the required annual return on investment per package? A. P 250 C. P1,200 B. P 600 D. P6,000 Horngren* 68. Rodenstock, Inc, currently places orders for a particular stock item at quarterly intervals. Information concerning these items is as follows: Cost of placing an order P10 Annual Demand 20,000 units Purchasing Price per unit P1.00 Carrying cost rate 10% What annual cost saving would result if Rodenstock used the EOQ for order sizes instead of their current policy? A. P80 C. P150 B. P90 D. P240 RPCPA 0592 69. Marlita works for a local ceramic company. She just completed her accountancy degree and learned the EOQ model in one of her subjects. She suggested to her employer to adopt it. The company sells 20,000 pcs of specialty ceramic items each year. Traditionally they have produced the items four times a year, making 5,000 pcs at a time. They carry no safety stock, as customers do not mind waiting for orders. The average piece of ceramic cost 400P400 to

Working Capital Management

make and costs the company P20 to carry in inventory for a year. The setup costs for each production run total P80. The company should A. Adopt EOQ due to savings of P35,675 B. Adopt EOQ due to savings of P42,320 C. Continue the existing system due to P38,950 advantage D. Continue the existing system due to P41,820 advantage RPCPA 0595 Questions 70 and 71 are based on the following information: CMA 0694 Melsie Computer Furniture Inc, (MCF) manufactures a line of office computer chairs. The annual demand for the chairs is estimated to be 5,000 units. The annual cost to hold one unit in inventory is P10 per year, and the cost to initiate a production run is P1,000. There are no computer chairs on hand, and MCF has scheduled four equal production runs of computer chairs for the coming year, the first of which is to be run immediately. RCF has 250 business days per year, sales occurred uniformly throughout the year, and production start-up is within one day. 70. The number of production runs per year of computer chairs that would minimize the sum of carrying and setup costs for the coming year is A. 1 C. 4 B. 2 D. 5 71. If RCF does not maintain a safety stock, the estimated total carrying costs for the computer chairs for the coming year based on their current schedule is A. P 4,000 C. P 6,250 B. P 5,000 D. P 12,500 Nos. 72 and 73 are based on the following information Roque, 2011 Using the EOQ model, Apple Baby Corporation computed the economic order quantity for one of the products it sells to be 4,000 units. Apple Baby Corporation maintains safety stock of 300 units. The quarterly demand for the product is 10,000 units. The order cost is ₱200 per order. The purchase price of the product is ₱2.40. The company sells at a 100% markup. The annual inventory carrying cost is equal to 25% of the average inventory level. 72. The total inventory order cost per year is A. ₱2,000 B. ₱2,300

C. ₱5,520 D. ₱800,000

73. The annual inventory carrying costs is A. ₱1,380 B. ₱2,300

C. ₱4,000 D. ₱4,300 Page 28 of 37

MANAGEMENT ADVISORY SERVICES

Questions 74 to 77 are based on the following information: Roque, 2011 The following information pertains to Emy Manufacturing Corporation’s Product X: Annual demand 33,750 units Annual cost to hold one unit of inventory ₱15 Setup cost (or the cost to initiate a production run) ₱500 Beginning inventory of product X 0 At present, the company produces 2,250 units of Product X per production run, for a total of 15 production runs per year. The company is considering to use the EOQ model to determine the economic lot size and the number of production runs that will minimize the total inventory carrying cost and setup cost for Product X. 74. If the EOQ model is used, the economic lot size is A. 1,500 units C. 1,500,000 B. 2,250 units D. 2,250,000 units 75. If the EOQ model is used, the number of production runs should be A. 15 runs C. 67.5 runs B. 22.5 runs D. 1,500 units 76. At present, the company’s total annual inventory costs is A. 7,500 C. 22,500 B. 16,875 D. 24,375 77. If the EOQ model is used, the total annual inventory costs, compared with that under present system, will increase (decrease) by A. (5,625) C. 3,750 B. (1,875) D. 11,250 78. Scholas Company uses 840,000 units of component R4 in manufacturing P444 over a 300day work year. The usual lead time for the part is six days, however at times the lead time has gone high as eight days. Scholas now desire to adjust its safety stock policy. The increase in safety stock is: A. 2,800 units C. 6,800 units B. 5,600 units D. 7,200 units RPCPA 0596 79. D & R Corporation consumes 300,000 units of spare part V per year. The average purchase lead time is 20 working days while maximum is 27 working days. The company’s annual operations cover 240 days allowing for shutdowns for plant maintenance, holidays and

Working Capital Management

Sundays. The company would like to keep safety stock or extra stock to guard against stock outs. How much is the safety stock? A. 1,250 units C. 25,000 units B. 8,750 units D. 33,750 units RPCPA 1096 80. Eklog Manufacturing Corporation uses the standard economic order quantity (EOQ) model. If the EOQ for Product A is 200 units and Eklog maintains a 50-unit safety stock for the item, what is the average inventory of Product A? A. 100 units C. 150 units B. 125 units D. 250 units CMA 1295 81. DF Tires unlimited is a business enterprise located in the City of Cagayan de Oro. The market price on a per unit basis is P 3,000. Since Cagayan de Oro is a very progressive rural place, the business sells an average of 36,000 tires annually. Based on a company study covering the last five years of its operations, it was found out that annual carrying cost per tire is P 5.00 and the ordering cost is P 100 on a per order basis. The store is open 7 days a week ( which includes Sundays and holidays of obligation). The delivery time per order (tires are ordered from Manila) is 5 days. Since it normally takes time before an order is placed, filled up and delivered, the manager has decided to keep a safety stock of 3,000 tires which is equivalent to a month’s sales. The average inventory is A. 3,493 tires C. 1,200 tires B. 3,600 tires D. 3,000 tires RPCPA 1091 82. For Raw Material L12, a company maintains a safety stock of 5,000 pounds. Its average inventory (taking into account the safety stock) is 12,000 pounds. What is the apparent order quantity? A. 6,000 lbs. C. 18,000 lbs. B. 14,000 lbs. D. 24,000 lbs. Bobadilla 83. Felix Company sells 200 discs per week. Purchase order lead time is 3 weeks and the economic order quantity is 450 units. What is the reorder point? A. 425 units C. 1,750 units B. 600 units D. 2,250 units RPCPA 0595 84. Huron Corporation purchases 60,000 headbands per year. The average purchase lead time is 20 working days, safety stock, equals 7 days normal usage and the corporation works 240 days per year. Huron should reorder headbands when the quantity in inventory reaches A. 1,750 units C. 5,250 units B. 5,000 units D. 6,750 units CMA 1293 Page 29 of 37

MANAGEMENT ADVISORY SERVICES

Working Capital Management

85. The China Tee Store sells 100,000 tea bags a year. Additional data are presented below: Selling price per bag 2.50 Purchase cost per bag 1.50 Ordering cost 5.40 an order Carrying cost 20% of unit cost Number of days the company operates in a year; 250 Average lead time on purchases; 6 days What is the reorder point if the company will keep a 10-day safety stock of inventory? A. 2,400 bags C. 6,400 bags B. 5,400 bags D. 8,800 bags RPCPA 0594 86. M & L Company has the following information on inventory: Sales 20,000 units per year Order Quantity 4,000 units Safety Stock 2,600 units Lead time 4 weeks What is the re-order point? (use 50 week year) A. 1,600 units C. 4,200 units B. 2,600 units D. 5,600 units

RPCPA 1096

Roque Maximum Lead Time Reorder Point

A. 15 days 1,050

B. 30 days 2,100

C. 45 days 2,100

D. 45 days 3,150

89. Canseco Enterprises uses 84,000 units of Part 256 in manufacturing activities over a 300-day work year. The usual lead-time for the part is 6 days; occasionally, however, the lead time has gone as high as 8 days. The company now desires to adjust its safety stock policy. The increase in safety stock size and the likely effect on stockout costs and carrying costs, respectively, would be CMA 1294 A. B. C. D. Increase in safety stock size 560 units 560 units 1,680 units 2,240 units Effect on stockout costs Decrease Increase Decrease Increase Effect on carrying costs Increase Decrease Increaase Decrease Questions 90 and 91 are based on the following information: The following information is available for Edgar Corporation’s Material X. Annual usage 12,600 units Working days per year 360 days Normal lead time 20 days The units of Material X are required evenly throughout the year.

Roque, 2011

87. Softdrinks Distributors, which buys in a pre-sell basis, is discussing with the route salesmen on the proper cases to be ordered and the frequency of call. From the route book and other records, the following are available: prior year’s purchases, 50,000 cases; carrying cost per case of inventory, P1.20; distributor’s discount, 1 case for every 10 cases bought; cost of placing and order, P3.00; weekly demand is approx. 952 cases. Safety stock required is 140 cases. No change in demand is expected this year. (Use 365 –day, 52-week). Determine the EOQ, and the reorder point assuming a two-day lead time. RPCPA 1094 a. b. c. d. EOQ 481 cases 500 cases 962 cases 250 cases Reorder point 500 cases 414 cases 275 cases 280 cases

90. What is the reorder point? A. 20th day B. 35 units

88. The following information pertains to Annie Corporation’s Material X: Annual usage 25,200 units Working days per year 360 days Normal lead time in working days 30 days Safety stock 1,050 units The maximum lead time in working days and the reorder point for material X are

Questions 92 thru 94 are based on the following information. CMA* Cantor Creations, which has 250 business days per year, manufactures desks for desktop workstations. The annual demand for the desks is estimated to be 5,000 units. The annual cost of carrying one unit in inventory is P10, and the cost to initiate a production run is P1,000. Cantor has scheduled four equal production runs for the coming year, the first to begin immediately. Currently, there are no desks on hand. Assume that sales occur uniformly throughout the year and that production is instantaneous.

C. 630 units D. 700 units

91. Assuming that occasionally, the company experiences delay in the delivery of Material X, such that the lead time reaches a maximum of 30 days, how many units of safety stock should the company maintain and what is the reorder point? A. B. C. D. Safety stock 0 350 350 1,050 Reorder point 1,050 700 1,050 700

Page 30 of 37

MANAGEMENT ADVISORY SERVICES

92. If Cantor Creations does not maintain a safety stock, the estimated total carrying costs for the desks for the coming year is A. P4,000. C. P6,250. B. P5,000. D. P10,250. 93. If Cantor Creations were to schedule only two equal production runs of the desks for the coming year, the sum of carrying costs and set-up costs would increase (decrease) by A. P(2,000). C. P4,250. B. P(250). D. P6,250. 94. A safety stock for a five-day supply of desks would increase the number of units in Cantor Creations’ planned average inventory by A. zero. C. 100. B. 50. D. 250. 95. Handy operated a chain of hardware stores across Laguna. The controller wants to determine the optimum safety stock levels for an air purifier unit. The inventory manager compiled the following data.  The annual carrying cost of inventory approximates 20% of the investment in inventory.  The inventory investment per unit averages P50  The stockout cost is estimated to be P5 per unit  The company orders inventory on the average of 10 times per year  Total cost = carrying cost + expected stockout cost The probabilities of a stockout per order cycle with varying levels of safety stock are as follows: Safety stock Stockout (Units) Probability 200 0 0% 100 100 15% 0 200 12% The total cost of safety stock on an annual basis with a safety stock level of 100 units is A. 550 C. 1,950 B. 1,750 D. 2,000 CMA 1294 96. Arnold Enterprises uses the EOQ model for inventory control. The company has an annual demand of 50,000 units for part number 191 and has computed an optimal lot size of 6,250 units. Per-unit carrying cost and stockout costs are P13 and P3, respectively. The following costs data have been gathered in an attempt to determine an appropriate safety stock level: Units Short Because of Number of times Short

Working Capital Management

Excess Demand during in the Last 40 The Lead Time Period Reorder Cycles 200 6 300 12 400 6 The annual cost of establishing a 200-unit safety stock is expected to be A. P2,600 C. P4,260 B. P4,040 D. P5,200

CMA 1288

97. Each stockout of a product sold by A.W. Inn Company costs P1,750 per occurrence. The carrying cost per unit of inventory is P5 per year, and the company orders 1,500 units of product 24 times a year at a cost of P100 per order. The probability of a stockout at various levels of safety stocks is. Units of safety stock Probability of stockout 0 .50 100 .30 200 .14 300 .05 400 .01 The optimal safety level for the company is A. 0 unit C. 300 units B. 100 units D. 400 units CMA 0692 98. Each stockout of Product AX sold by Axiom Inc costs P8,750 per occurrence. The carrying cost per unit of inventory is P250 per year and the company orders 1,500 units of product 24 times a year at a cost of P5,000 per order. The probability of stockout at various levels of safety stock is Units of safety stock Probability of stockout 0 .50 100 .30 200 .14 300 .05 400 .01 The optimal safety stock level for the company is A. 0 unit C. 200 units B. 100 units D. 300 units RPCPA 0596* 99. Paeng Company uses the EOQ model for inventory control. The company has an annual demand of 50,000 units for part number 6702 and has computed an optimal lot size of 6,250 Page 31 of 37

MANAGEMENT ADVISORY SERVICES

units. Per-unit carrying costs and stockout costs are P9 and P4, respectively. The following data have been gathered in an attempt to determine an appropriate safety stock level: Units Short Because of Excess Number of Times Short Demand during the Lead in the last 40 Reorder Time Period Cycles 100 8 200 10 300 14 400 8 What is the optimal safety stock level? A. 100 units C. 300 units B. 200 units D. 400 units Bobadilla Short-term Financing 100. Batchoy & Company buys on terms 2/10, net 30, but generally does not pay until 40 days after the invoice date. Its purchases total P2,160,000 per year. Assuming 360 days a year, the amount of “non-free” trade credit used by the company on the average. A. P60,000 C. P180,000 B. P120,000 D. P240,000 RPCPA 0595 101. If a firm purchases raw materials from its suppliers on a 2/10, n/60 cash discount basis, the equivalent annual interest rate (using a 360-day year) of foregoing the cash discount and making payment on the 60th day is A. 12.2% C. 36.7% B. 14.7% D. 73.5% RPCPA 0596 102. If a firm purchases raw materials from its supplier on a 2/10, net 40, cash discount basis, the equivalent annual interest rate (using 360-day year) of foregoing the cash discount and making payment on the 40th day is A. 2% C. 24.49% B. 18.36% D. 36.72% Gleim 2013 103. Paramount Corporation is offered trade credit terms of 3/15,net 45. The firm does not take advantage of the discount, and it pays the account after 67 days. Using a 365-day year, what is the nominal annual cost of not taking discount? A. 18.2% C. 23.48% B. 21.71% D. 26.45% Wiley 2012 104. Escape Company regularly pays its accounts payable on the tenth day and enjoys the 2

Working Capital Management

percent cash discount, term: 2/10, net 30. Because of an oversight, one supplier’s invoice is not paid within the discount period but paid 10 days after. What is the annual cost of that incident of paying an invoice on the 20th day, instead of the tenth day? Use 360 days a year. A. 2.04% C. 36.73% B. 30.02% D. 73.44% Bobadilla 105. Software Center Inc’s new controller is reviewing the company’s cash management. Below are relevant information regarding trade credits from the suppliers of the company: Supplier Average Monthly Purchases Credit terms Tech Co. ₱ 100,000 Net 30 Computech ₱ 300,000 2/10, n/30 Compuworks ₱1,000,000 5/10, net 120 So-wares ₱ 600,000 3/10, net 45 The company uses a 360-day year. Assume that all supplier can supply any and all of the requirements of software and can provide unlimited credit line to the company and the company can only have one supplier. With a cost of bank borrowing of 18% per annum, which supplier would Software choose? A. Tech Co. due to no discount policy. B. Computech due to the trade credit of 36.7%. C. Compuworks due to the highest trade discount at 5%. D. Compuworks due to the longest credit term of 120 days. RPCPA 1096 Questions 106 and 107 are based on the following information. CMA 1296 CyberAge Outlet, a relatively new store, is a cafe that offers customers the opportunity to browse the internet or play computer games at their tables while they drink coffee. The customer pays a fee based on the amount of time spent signed on to the computer. The store also sells books, tee shirts, and computer accessories. CyberAge has been paying all of its bills on the last day of the payment period, thus forfeiting all supplier discounts. Shown below are data on CyberAge’s two major vendors, including average monthly purchases and credit terms. Vendor Ave. Monthly Purchases Credit Terms Web Master P25,000 2/10, n/30 Softidee 50,000 5/10, n/90 106. Assuming a 360-day and that CyberAge continues paying on the last day of the credit period, the company’s weighted average annual interest rate for trade credit (ignoring the effects of compounding) for these two vendors is A. 25.2% C. 28.0% B. 27.0% D. 30.2% Page 32 of 37

MANAGEMENT ADVISORY SERVICES

107. Should CyberAge use trade credit and continue paying at the end of the credit period? A. Yes, if the cost of alternative short-term financing is less. B. No, if the cost of alternative long-term financing is greater. C. Yes, if the cost of alternative short-term financing is greater. D. Yes, if the firm’s weighted average cost of capital is equal to the weighted average cost of trade credit. 108. A one year, P20,000 loan with a 10% nominal interest rate provides the user with the use of if interest is charged on a basis. CIA 0595 A. B. C. D. List A P18,000 P20,000 P20,000 P22,000 List B Simple Simple Discount Discount 109. If the firm borrows P185,000 at 8 percent on a one-year discounted loan, what is the effective interest rate? A. 7.41 percent C. 8.70 percent B. 8.00 percent D. 9.07 percent Bobadilla 110. Hager Company’s bank requires a compensating balance of 20% on a P100,000 loan. If the stated interest on the loan is 7%, what is the effective cost of the loan? A. 5.83% C. 7.00% B. 6.40% D. 8.75% CMA 0697 111. Butuan Company recently received a commercial bank loan of 16% discounted rate with a 20% compensating balance. The term of the loan is one year. The effective cost of borrowing is: A. 19.05% C. 22.05% B. 20.00% D. 25.00% RPCPA 0596 112. The Peninsula Commercial Bank and Island Corporation agreed to the following loan proposal: Stated interest rate of 10% on a one-year discounted loan; and 15% of the loan as compensating balance on zero-interest current account to be maintained by Island Corporation with Peninsula Commercial Bank. The loan requires a net proceeds of P1.5 million. What is the principal amount of loan applied for as part of the loan agreement? A. P1,125,000 C. P1,764,706 B. P1,666,667 D. P2,000,000 Bobadilla 113. A short-term bank loan will have a higher effective financing cost if it has which combination

Working Capital Management

of characteristics? A. A 10% compensating balance and regular interest B. A 20% compensating balance and regular interest C. A 10% compensating balance and discount interest D. A 20% compensating balance and discount interest

CIA 1196

114. Cool and Sweet obtained a short-term bank loan for P1 Million at an annual interest of 12%. As a condition of the loan, the company is required to maintain a compensating balance of P200,000 in its savings account which earns at an annual rate of 6%. The company would otherwise maintain only P100,000 on the savings account for transactional purposes. The effective cost of the loan is: A. 12% C. 13.20% B. 12.67% D. 13.5% RPCPA 1095 115. Echo borrowed P100,000 from a bank on a one-year 8% term loan, with interest compounded quarterly. What is the effective annual interest on the loan? A. 8% C. 9.12% B. 8.24% D. 10.41% Wiley 2012 116. On January 7, 2006, Dean Company discounted its own P100,000, 180-day note at United National Bank at a discount rate of 20%. Dean repaid the note on the July 6, 2005, due date. Based on a 360-day year, the effective rate of interest on the borrowing was A. 18.2% C. 22.2% B. 20.0% D. 25.0% Agamata 2013 117. What is the effective rate of a 15% discounted loan for 90 days, P200,000, with 10% compensating balance? Assume 360 days per year. A. 15.0% C. 20.0% B. 17.4% D. 22.2% Bobadilla 118. Perlas Company borrowed from a bank an amount of P1,000,000. The bank charged a 12% stated rate in an add-on arrangement, payable in 12 equal monthly installments. What is the estimated annual effective rate? A. 12.70% C. 24.00% B. 22.15% D. 25.05% Bobadilla 119. Nardo, Inc. can issue a three-month commercial paper with a face value of P1,000,000 fpr P980,000. Transaction costs would be P1,200. The annualized percentage cost of the financing would be Page 33 of 37

MANAGEMENT ADVISORY SERVICES

A. 2.17% B. 8.00%

Working Capital Management

C. 8.48% D. 8.67%

Cabrera

120. New Town Bank offers a P25,000 line of credit with an interest rate of 2.5 percent per quarter. The loan agreement also requires that 5 percent of the unused portion of the credit line be deposited in a non-interest bearing account as a compensating balance. Short-term investments are currently paying 1.6 percent per quarter. Assume any funds borrowed or invested use compound interest. What is the effective annual interest rate on the line of credit if a customer borrows the entire P25,000 for one year? A. 6.56% C. 9.87% B. 8.98% D. 10.38% Bobadilla 121. The Friendly Bank offers AB United a P200,000 line of credit with an interest rate of 2.25 percent per quarter. The credit line also requires that 2 percent of the unused portion of the credit line be deposited in a non-interest bearing account as a compensating balance. AB United's short-term investments are paying 1.5 percent per quarter. Assume any funds borrowed or invested use compound interest. What is the effective annual interest rate on this arrangement if the line of credit goes unused all year? A. 5.92% C. 6.08% B. 6.00% D. 6.14% Bobadilla 122. The Clay Company has a revolving line of credit of P300,000 with a one-year maturity. The terms call for a 6% interest rate and a ½ percent commitment fee on the unused portion of the credit line. The average loan balance during the year was P100,000. The annual cost of this financing arrangement is A. P6,000 C. P7,000 B. P6,500 D. P7,500 CMA Samp 123. The treasury analyst for KG Manufacturing has estimated the cash flows for the first half of the next year (ignoring any short term borrowings) as follows: Cash (millions) Inflows Outflows January P2 P1 February 2 4 March 2 5 April 2 3 May 4 2

June 5 3 KG has a line of credit up to P4million on which it pays interest monthly at a rate of 1% of the amount utilized. KG is expected to have a cash balance of P2million on January 1 and no amount utilized on its line of credit. Assuming all cash flows occur at the end of the month, approximately how much will KG pay in interest during the first half of the year? A. Zero C. P61,000 B. P 50,000 D. P 132,000 CMA 0697 Questions 124 thru 126 are based on the following information. Bobadilla Caylor Inc. needs to borrow P300,000 for the next 6 months. The company has a line of credit with a bank that allows the company to borrow funds with a 10% interest rate subject to a 25% of loan compensating balance. Currently, Caylor Inc. has no funds on deposit with the bank and will need the loan to cover the compensating balance as well as their other financing needs. 124. How much will Caylor Inc. need to borrow? A. P225,000 B. P330,000

C. P375,000 D. P400,000

125. What will be the annual percentage rate, or APR, for this financing? A. 10.00% C. 12.12% B. 10.67% D. 13.33% 126. What is the annual percentage rate for this financing scheme assuming the interest is discounted? A. 13.98% C. 16.67% B. 14.29% D. 20.00% 127. A company enters into an agreement with a firm that will factor the company’s accounts receivable. The factor agrees to buy the company’s receivables, which average P100,000 per month and have an average collection period of 30 days. The factor will advance up to 80% of the face value of receivables at an annual rate of 10% and a charge fee of 2% on all receivables purchased. The controller of the company estimates that the company would save P18,000 in collection expenses over the year. Fees and interest are not deducted in advance. Assuming a 360-day year, what is the annual cost of financing? A. 10.0% C. 14.0% B. 12.0% D. 17.5% CMA 0696 128. A firm often factors its account receivable. Its finance company requires a 6% reserve and charges a 1.4% commission on the amount of the receivables. The remaining amount to be Page 34 of 37

MANAGEMENT ADVISORY SERVICES

Working Capital Management

advanced is further reduced by an annual interest charge of 15%. What proceeds (rounded to the nearest peso) will the firm receive from the finance company at the time a P100,00 account due in 60 days is factored? A. P92,600 C. P90,285 B. P96,135 D. P85,000 Gleim 2013 Questions 129 to 132 are based on the following information: Roque 2013 Lei Company enters into an agreement with a firm that will buy Lei Company’s accounts receivable and assume the risk of collection. Details about the agreement are as follows. Average amount of receivable to be factored each month: P500,000 Average collection period: 60 days Amount to be advanced by the factor : 80% of the face amount of the receivables Interest rate, deductible in advance: 10% p.a. Factor’s fee, deductible in advance: 2% Annual savings of Lei Company in collection expenses: P60,000 129. How much is the monthly net proceeds from factoring the receivables? A. P350,000 C. P400,000 B. P383,333 D. P500,000 130. What is the annual net cost of factoring? A. (P10,000) B. P100,000

C. P120,000 D. P160,000

131. What is the effective annual cost rate of financing? A. 20% C. 26.09% B. 25% D. 29.41% 132. If the interest charge and factor’s fee is not deducted in advance, the effective annual cost rate is A. 20% C. 26.09% B. 25% D. 29.41% 133. An invoice of a P100,000 purchase has credit terms of 1/10, n/40. A bank loan for 8 percent can be arranged at any time. When should the customer pay the invoice? A. Pay on the 1st. C. Pay on the 40th B. Pay on the 10th D. Pay on the 60th Bobadilla

134. The high cost of short-term financing has recently caused a company to re-evaluate the terms of credit it extends to its customers. The current policy is 1/10, net 60. If customers can borrow at prime rate, at what prime rate must the company change its terms of credit in order to avoid an undesirable extension in its collection of receivables? A. 2% C. 7% B. 5% D. 8% Gleim 2013 135. Three suppliers of Mama Corporation offer different credit term. Core Co. offers term of 1 1/2/15, net 30. Doug Corp offers terms of 1/10, net 30. Ernst Inc. offers of 2/10, net 60. Mama Corporation would have to borrow at a bank at an annual rate of 12% in order to take any cash discounts. Which one of the following would be the most attractive for Ma Corp.? (Assume 360 days in a year) A. Purchase from Doug Corp and pay in 30 days. RPCPA 0595 B. Purchase from Core Co., pay in 15 days and borrow any money needed from the bank. C. Purchase from Core Co., pay in 30 days and borrow any money needed from the bank. D. Purchase from Ernst Inc., pay in 60 days and borrow any money needed from the bank. 136. Buddah Corporation intends to acquire a new equipment to increase its capacity. It is estimated to cost P2.4 million. A bank loan can finance the acquisition at ten percent (10%) discounted interest. Alternatively, the company may adjust delay payment to its suppliers. Presently, the company buys under terms of 2/10, net 40. But management believes payment could be delayed 30 additional days, without penalty, that is, payment could be made in 70 days. Assuming 360 days a year, the company should... A. Delay payments to suppliers since it does not cost anything. B. Borrow since it is cheaper by 2.5% than delaying payment to suppliers. C. Borrow since it is cheaper by 1.13% than delaying payment to suppliers. RPCPA 0597 D. Delay payments to suppliers since it would cost 12% as against bank loan of 10%. 137. Meals Etc has been very successful. It is the newest fast food outlet at the Greenbelt of Makati featuring ordinary Filipino food packed with banana leaves. After six months of operations, it needs to expand. The owner, Mr. K Eng, estimates that P2.4Million will be required to put up another outlet in Ortigas area. Financing was offered by a friendly banker at 10% discounted interest. Alternatively, Mr, Eng is thinking of just delaying payment to its suppliers. All his sales are on cash basis. The company purchases under terms of 2/10, net.40 but Mr. Eng believes that he could delay payments by another 30 days without any problem. This means payment could be made in 70 delays. Assuming 360 days a year, Meals Etc should opt for A. Bank loan since it costs of 10% is cheaper than the cost of delaying payments of 12%. Page 35 of 37

MANAGEMENT ADVISORY SERVICES

B. Delaying payments since it costs only 2% compared to 10% discounted bank interest. C. Delaying payments since it has no cost compared to the 10%discounted bank interest. D. Bank loan since its cost of 11.11% is cheaper than the cost of delaying payments of 12.24%. RPCPA 0597 138. A corporation is currently experiencing cash-flow problems and has determined that it is in need of short-term credit. It can either use its trade credit on P100,000 of accounts payable with terms of 1/10, net 30 or a 30-day note with a 20% annual simple interest rate. Which is the best alternative, and what is its effective rate of interest (rounded to a whole percentage and using a 360-day year)? A. The note. Its effective rate is 17% B. The note. Its effective rate is 20% C. The trade credit. Its effective rate is 10% D. The trade credit. Its effective rate is 20% CIA 1188 139. Jun Traders, a merchandising firm, purchases merchandise from its suppliers on credit terms of 2/10, net 30. Jun Traders needs cash, so it is considering two alternatives: Alternative 1 – Obtain a short-term loan from a bank at an effective interest rate of 12%. Alternative 2 – Forego the discount on its credit purchases and pay on the 30th day of the term. Jun Traders should choose (Use a 360-day year.) A. Alternative 1 because its cost is cheaper by 1%. B. Alternative 2 because its cost is cheaper by 10%. C. Alternative 1 because its cost is cheaper by 24.73%. D. Alternative 2 because this is a costless credit financing. Roque, 2013 140. Every 15 days a company receives P10,000 worth of raw materials from its suppliers. The credit terms for these purchases are 2/10, net 30, and payment is made on the 30th day after each delivery. Thus, the company is considering a 1-year bank loan for P9,800 (98% of the invoice amount). If the effective annual interest rate on this loan is 12%, what will be the net peso savings over the year by borrowing and then taking the discount on the materials? A. P1,176 C. P3,624 B. P1,224 D. P4,800 Bobadilla 141. A company has accounts payable of P5 million with terms of 2% discount within 15 days, net 30 days (2/15 net 30). It can borrow funds from a bank at an annual rate of 12%, or it can wait until the 30th day when it will receive revenues to cover the payment. If it borrows funds on the last day of the discount period in order to obtain the discount, its total cost will be A. P 24,500 more C. P 75,500 less

Working Capital Management

B. P 51,000 less

D. P100,000 less

Bobadilla

142. A company will receive cash from sales in 1 year that can be used to pay for materials. The supplier will allow payment in 1 year. If the company pays the supplier immediately, it will receive a 20% discount off the P100,000 purchase price, but it must borrow the full amount. A bank has offered the company three alternatives:  A 1-year loan at 18% with no other fees  A 1-year loan at 15% with the provision that it maintains 20% of whatever amount it borrows as noninterest-bearing compensating balances over the life of the loan, or  A guaranteed line of credit of P100, 000 at 17% with the provision that the bank will collect a 1% fee on the average amount of unused funds. The company expects to borrow no other funds The company would achieve the lowest cost of financing by: A. Accepting the 1 year loan at 18% with no other provisions. B. Accepting the 1 year loan at 15% with compensating balance provisions. C. Allowing the supplier to finance the materials and making payment at the end of 1 year. D. Accepting the guaranteed line of credit at 17% with the fee required on the average amount of unused funds CIA 1186 Questions 143 to 146 are based on the following information: Roque 2013 Jem Traders, Inc. needs P100,000 to pay a supplier’s invoice for merchandise purchased with terms 2/10, net 30. Jem Traders want to pay on the 10th day of the credit term so it can avail of the 2% discount. The funds needed can be raised by obtaining a short-term loan from a bank which agrees to grant a 30-day loan at 12% discounted interest per annum. The bank requires that a compensating balance of 10% be maintained in the borrower’s non-interest earning deposit account. 143. The amount needed by Jem Traders to pay the invoice within the discount period is A. P9,000 C. P100,000 B. P98,000 D. P102,000 144. The principal amount of the loan that must be obtained from the bank to raise the needed fund is A. P108,780 C. P112,360 B. P110,112 D. P125,640 145. What is the effective interest rate of the loan? A. 10% C. 13.48% Page 36 of 37

MANAGEMENT ADVISORY SERVICES

B. 12%

Working Capital Management

D. 22%

146. If Jem Traders fails to pay the discount and pays the account on the 30th day of the term, what is the annual cost of this non-free trade credit? A. 0 C. 24% B. 2% D. 36.73% Questions 147 thru 149 are based on the following information: Roque 2013 The expected boom in business in the coming period led the Baby Apple Company to decide to expand its operations. The expansion requires an increase of P500,000 in working capital, which the company is considering to finance through any of the following alternatives: 1. Pledge the accounts receivable. The company’s average accounts receivable is P625,000 per month. A financer will lend 80% of the face value of the receivables at 10% interest per annum, payable on the maturity loan. 2. Issue P515,000 of 3-month commercial paper to net P500,000. New paper will be issued every 3 months. 3. Borrow from a commercial bank an amount that will net P500,000 after deducting a compensating balance of 15% and interest of 5%. Use a 360-day year in all your calculations. 147. The cost of Alternative 1 is A. 8% B. 10%

C. 12.5% D. 120%

148. The annual cost of Alternative 2 is A. 0.97% B. 1%

C. 11.65% D. 12%

149. The annual cost of Alternative 3 is A. 5% B. 6.25%

C. 20% D. 25%

on all receivables purchased. The controller estimates that the firm would save P24,000 in collection expenses over the year. Assume the fee and interest are not deductible in advance. Alternative (B): Borrow P110,000 from a bank at 12% interest. A 9% compensating balance would be required. Alternative (C): Issue P110,000 of 6-month commercial paper for P100,000 (New paper would be issued every 6 months) Alternative (D): Borrow P125,000 from a bank on a discount at 20%. No compensating balance would be required. Assume a 360-day year in all of your calculations. 150. The cost of alternative A is A. 10.0% B. 12.0%

C. 13.2% D. 16.0%

151. The cost of alternative B is A. 9.0% B. 12.0%

C. 13.2% D. 21.0%

152. The cost of alternative C is A. 9.1% B. 10.0%

C. 18.2% D. 20.0%

153. The cost of alternative D is A. 20.0% B. 25.0%

C. 40.0% D. 50.0%

Questions 150 to 153 are based on the following information. CMA 1296 The Frame Supply Company has just acquired a large account and needs to increase its working capital by P100,000. The controller of the company has identified the four sources of funds given below: Alternative (A): Pay a factor to buy the company’s receivables, which average P125,000 per month and have an average collection period of 30 days. The factor will advance up to 80% of the face value of receivables at 10% interest and charge a fee of 2% Page 37 of 37